You are on page 1of 144

Image-Based Case Studies

in
ENT and Head & Neck Surgery
Image-Based Case Studies
in
ENT and Head & Neck Surgery

Rahmat Omar MBBS (Malaya) MS ORL (UM)


Consultant Otolaryngologist and
Head and Neck Surgeon
Department of Otorhinolaryngology
Faculty of Medicine
University of Malaya, Kuala Lumpur, Malaysia

Prepageran Narayanan
MBBS (Malaya) MS ORL (UM) FRCS (Ed) FRCS (Glas)
Professor
Senior Consultant Otolaryngologist and
Head and Neck Surgeon
Department of Otorhinolaryngology
Faculty of Medicine
University of Malaya
Kuala Lumpur, Malaysia

Foreword
Ghauth Jasmon

JAYPEE BROTHERS MEDICAL PUBLISHERS (P) LTD


New Delhi Panama City London Dhaka Kathmandu

Jaypee Brothers Medical Publishers (P) Ltd.

Headquarters
Jaypee Brothers Medical Publishers (P) Ltd
4838/24, Ansari Road, Daryaganj
New Delhi 110 002, India
Phone: +91-11-43574357
Fax: +91-11-43574314
Email: jaypee@jaypeebrothers.com

Overseas Offices
J.P. Medical Ltd. Jaypee-Highlights Medical Publishers Inc.
83 Victoria Street, London City of Knowledge, Bld. 237, Clayton
SW1H 0HW (UK) Panama City, Panama
Phone: +44-2031708910 Phone: 507-301-0496
Fax: +02-03-0086180 Fax: + 507-301-0499
Email: info@jpmedpub.com Email: cservice@jphmedical.com
Jaypee Brothers Medical Publishers (P) Ltd Jaypee Brothers Medical Publishers (P) Ltd
17/1-B Babar Road, Block-B, Shaymali Shorakhute, Kathmandu
Mohammadpur, Dhaka-1207 Nepal
Bangladesh Phone: +00977-9841528578
Mobile: +08801912003485 Email: jaypee.nepal@gmail.com
Email: jaypeedhaka@gmail.com

Website: www.jaypeebrothers.com
Website: www.jaypeedigital.com

2013, Jaypee Brothers Medical Publishers

All rights reserved. No part of this book may be reproduced in any form or by any means
without the prior permission of the publisher.

Inquiries for bulk sales may be solicited at: jaypee@jaypeebrothers.com

This book has been published in good faith that the contents provided by the authors contained
herein are original, and is intended for educational purposes only. While every effort is made
to ensure accuracy of information, the publisher and the authors specifically disclaim any
damage, liability, or loss incurred, directly or indirectly, from the use or application of any
of the contents of this work. If not specifically stated, all figures and tables are courtesy of
the authors. Where appropriate, the readers should consult with a specialist or contact the
manufacturer of the drug or device.

Image-Based Case Studies in ENT and Head & Neck Surgery

First Edition: 2013

ISBN 978-93-5090-089-5

Printed at
Dedicated to
My beloved parents:
for your everlasting inspirations and motivations.
My family: Dr Salmah Ismail, Nur Rabbaniyyah, Muhammad Iqbal,
Ahmad Farhan, and Mursyid Syaqeer.
My students, past and present.
Rahmat Omar

The Divine Mother and My Family


Prepageran Narayanan
Foreword

It gives me a great pleasure to write a few words on the Image-Based Case


Studies in ENT and Head & Neck Surgery.
I understand that this is the second book of this nature published by the
authors; the successor of the first book which was an outstanding success.
This proves that there is a demand for clear precise pictorial reference books
as it enables the readers to appreciate the varied clinical presentations and
its subsequent management in day-to-day clinical practice. This questions
and answers book will complement the previous book which was an atlas.
It is said a picture paints a thousand words and this is indeed true in clinical
settings. This book covers the varied clinical images in different segments of
otorhinolaryngology (ENT), ranging from otology, rhinology, head and neck
and laryngology. It is a testament to the dedication and enthusiasm of the
authors. I wish to congratulate the authors Associate Professor Dr Rahmat
Omar and Professor Dr Prepageran on the publication of this book.
University of Malaya is committed to become a center of excellence in
academic pursuit not only in Malaysia but also in the region and internationally.
We encourage knowledge expansion and dissemination, and books written by
young upcoming academicians are very much an integral part of the University
of Malayas effort to promote academics and culture of publishing among its
academic staffs. I hope this will encourage more authors to put their wisdom
and knowledge gathered over the years into books of similar nature to keep
the readers updated and refreshed with core as well as current best practice
information.
Finally, I would like to compliment the authors again and believe this
book will be well accepted among a wide spectrum of readers ranging from
medical students, primary care physicians to ENT masters students.

Ghauth Jasmon
Vice Chancellor
University of Malaya
50603 Kuala Lumpur
Preface

It is our pleasure to write a questions and answers book in ENT and Head
and Neck Surgery which seems appropriate for the purpose of exercise and
quick revision especially to our undergraduate and postgraduate students. The
fundamental will still be the core of this book with applied and functional
knowledge added for integration. These are based on real scenarios and clinical
findings which were encountered during daily outpatient practice and in the
operating table during surgery. The cases were carefully selected to represent
the main issue of particular question.
Almost all the images were taken and edited by the first author. The
book is divided into four sections; Otology, Rhinology, Head and Neck, and
Laryngology. Special techniques like endoscopy and videostroboscopy were
used to enable excellent high quality color photographs to be taken especially
from the ear, nasal cavity, and larynx.
This book is complementary to the Ear Nose Throat Colour Atlas and
Synopsis which was published earlier by University of Malaya Press. It would
be an ideal resource for exam preparation and revision; hopefully readers
would find it useful and practical. For qualified specialists and ENT surgeons,
this book would be an indispensable quick reference.
We would like to thank the patients appearing in this book, as without
their willingness and cooperation this work would not have been completed.
Special thanks to the medical officers in training and colleagues from the
Department of Otorhinolaryngology, University of Malaya, who had directly
or indirectly contributed in any way in the production of this book.

Rahmat Omar
Prepageran Narayanan
Contents

1. Otology................................................................................................. 1
(Q. 1 to 43)

2. Rhinology............................................................................................ 44
(Q. 44 to 76)

3. Head and Neck................................................................................... 80


(Q. 77 to 102)

4. Laryngology....................................................................................... 105
(Q. 103 to 124)
Abbreviations

AA : atticoantral
BCC : basal cell carcinoma
BERA : brainstem evoked response audiometry
CA : carcinoma
CaHA : calcium hydroxyapatite
CAT : combined approach tympanoplasty
CO2 laser : carbon dioxide laser
CP : cerebellopontine angle
CSF : cerebrospinal fluid
CSOM (AA) : chronic suppurative otitis media (atticoantral)
CSOM (TT) : chronic suppurative otitis media (tubotympanic)
CT scan : computed tomography-scan
DCR : dacryocystorhinostomy
DNS : deviated nasal septum
ET : Eustachian tube
FBC : full blood count
FNAC : fine needle aspiration cytology
FOR : fossa of Rosenmller
GA : general anesthesia
HPE : histopathological examination
HRCT : high resolution computed tomography
IAM : internal auditory meatus
LPR : laryngopharyngeal reflux
MEE : middle ear effusion
MRI : magnetic resonance imaging
MRM : modified radical mastoidectomy
M&G : myringotomy and grommet
OMC : osteomeatal complex
PNS : postnasal space
PTA : pure tone audiogram
SNHL : sensorineural hearing loss
TB : tuberculosis
TM : tympanic membrane
TT : tubotympanic
VS : vestibular schwannoma
1 Otology

Questions

Q. 1. What does this picture show?

Q. 2. a. What does this otoendoscopic examination show?


b. Why does this occur?
c. What are the possible causes?
Image-Based Case Studies in ENT and Head & Neck Surgery

Q. 3. a. What procedure was taken as demonstrated in this picture?


b. What are the indications?
c. What are the considerations when performing the
procedure and its potential complications?

Q. 4. A middle-aged lady presented with reduced hearing affecting the


right ear associated with the past history of recurrent ear discharge
since childhood. Otoendoscopic examination and tuning fork
test was performed.
a. Describe the findings.
b. What would be the expected tuning fork test result?
c. How this patient would be managed further?
Q. 5. This is an image of the right ear.
a. What does it show?
b. What is the management?

Otology
(Abbreviations: A, anterior; P, posterior)

Q. 6. a. What is the abnormality seen?


b. What are the possible complications and the subsequent
management?
Image-Based Case Studies in ENT and Head & Neck Surgery

Q. 7. a. What is the diagnosis?


b. Explain the underlying pathology and the subsequent
management.

Q. 8. a. What does this picture show?


b. How would this patient be managed further?
A

Otology
5

B
Q. 9. A male teenager presented with a painful left ear for 2 days
duration followed by itchiness and inability to close the eye on
the same side.
a. Describe the findings as shown.
b. What is the diagnosis?
c. Outline further management of this patient.
Image-Based Case Studies in ENT and Head & Neck Surgery

Q. 10. This is an otoendoscopic view of a patient who presented with


the history of reduced hearing.
a. What is the diagnosis?
b. What are the other common clinical features?
c. How would this child be managed further?

Q. 11. a. What does this examination of a childs ear show?


b. How is this condition managed?
Q. 12. This patient presented with an itchy painful ear.
What is the diagnosis, etiology and the subsequent treatment?

Otology
7

Q. 13. This patient presented with recurrent ear discharge.


What is the diagnosis, immediate and long-term management?
Image-Based Case Studies in ENT and Head & Neck Surgery

Q. 14. This patient presented with prolonged ear discharge. Otoscopic


findings were as shown.
a. What is the diagnosis?
b. How is the presentation different from chronic suppurative
otitis media (tubotympanic)?
c. How would this condition be managed further?

Q. 15. a. What does this otoscopy picture show?


b. What are the possible causes?
c. How is this condition managed?
Q. 16. a. What does this picture reveal and what operation has been
performed?
b. What are the potential complications related to the surgery?

Otology
c. What are the alternative procedures that can be carried
out and in what condition?

Q. 17. A 25-year-old gentleman presented with hearing loss as the only


symptom. What is the diagnosis and its subsequent management?
Image-Based Case Studies in ENT and Head & Neck Surgery

Q. 18. This is an otoscopic finding of a patient who presents with fullness


of ear and mild hearing loss.
a. What does the otoendoscopy reveal?
b. Why is the hearing loss minimal?
c. What can we do further?

10

Q. 19. What is the pathology seen and what would be the next course
of action in this patient with hearing loss?
Q. 20. a. What does this picture show?
b. What would have contributed to this appearance?

Otology
11

Q. 21. This is the image of an ear canal.


a. What does this picture show?
b. How would this condition be managed further?
Image-Based Case Studies in ENT and Head & Neck Surgery

Q. 22. This patient presented with bleeding from the left ear.
a. What are the possible diagnoses?
b. Outline the subsequent management.

12

Q. 23. a. What is this image?


b. Name the parts that are numbered.
A

Otology
13

B
Q. 24. a. What does this image show?
b. What are the differential diagnoses and how do you differentiate
them?
c. What are the clinical features?
d. Outline the subsequent management.
Image-Based Case Studies in ENT and Head & Neck Surgery

Q. 25. This is an otoendoscopic view of a middle-aged female who


complains of an ear blockage.
What do you see and the probable underlying pathology?

14

Q. 26. This child is undergoing a hearing assessment.


What is the name of the test and its indications?
A

Otology
15

B
Q. 27. A 45-year-old lady complained of right-sided hearing loss, which
has become worse since her last pregnancy. Examination revealed
a normal tympanic membrane and the tuning fork tests showed
a moderately severe conductive hearing loss. She underwent
surgery and reconstructive procedure was performed.
a. What is the diagnosis?
b. Describe the findings as shown in Figures (A) and (B)?
c. What procedure would probably have been performed?
Image-Based Case Studies in ENT and Head & Neck Surgery

16

B
Q. 28. A 29-year-old lady was involved in a road traffic accident and
sustained intracranial injury. She was referred to the ENT team
few days later for the complaint of reduced hearing affecting the
left side. Otological examination revealed above findings.
a. What is the clinical sign demonstrated in Figure (A) and on
otoscopic evaluation in Figure (B)?
b. What other signs should be looked for?
c. How would this patient be managed further?
A

Otology
17

B
Q. 29. This child presented in the clinic with the above findings.
What is your diagnosis and subsequent management?
Image-Based Case Studies in ENT and Head & Neck Surgery

Q. 30. This is the audiogram of a patient who presented with bilateral


tinnitus. What is the diagnosis and further management?

18
A

Otology
19

B
Q. 31. A 69-year-old lady presented with pulsatile tinnitus involving
the left ear associated with mild decrease in hearing. Otoscopic
examination and subsequent imaging study was ordered.
a. Describe the otoscopic findings.
b. What does the imaging study show?
c. Give the possible diagnoses.
d. Outline the treatment.
Image-Based Case Studies in ENT and Head & Neck Surgery

Q. 32. A 37-year-old man was involved in motorcycle accident and


sustained head injury needing temporary intensive care. He was
referred for ENT evaluation and further assessment.
a. Describe the imaging study findings.
b. What is the diagnosis?
c. List the possible complications from this lesion.
d. How should this patient be managed further?
20
A

Otology
21

B
Q. 33. This child had surgery done for profound hearing loss. Related
intraoperative images were as shown.
a. What surgery was performed?
b. List its indications.
c. What are the necessary preoperative evaluations needed to
be done?
d. How would this child be monitored postoperatively?
Image-Based Case Studies in ENT and Head & Neck Surgery

Q. 34. A 17-year-old teenager presented with an increasingly painful left


ear swelling of 5 days duration. She admitted to have scratched
the pinna due to itchiness, but denied having any recent injury or
ear piercing procedure done.
a. Describe the findings.
b. Give the most likely diagnosis.
c. How would this lesion be managed further?
22 d. List potential complications that can occur.
Q. 35. A patient presented to outpatient otolaryngology clinic with complains
of painful left ear associated with humming sounds. Further history
revealed he had quarreled with a friend and ended with a fight.

Otology
a. Describe the otoendoscopic findings.
b. What are the roles of hearing test and audiometry in this instance?
c. How would you tell the patient regarding the prognosis?

23
Image-Based Case Studies in ENT and Head & Neck Surgery

24

B
Q. 36. A 14-year-old teenager presented with longstanding history of
reduced hearing affecting the left ear. Tuning forks test showed
negative Rinnes test on the left with ipsilateral lateralization on
Webers test. X-ray was ordered.
a. Descibe the radiological findings.
b. What is the most likely diagnosis?
c. List its potential complications.
d. How would this patient be managed further?
Q. 37. This intraoperative image was taken during a myringoplasty.
a. Describe the step currently being taken.
b. What is its role in myringoplasty?
c. List other materials that can be used for similar purpose.

Otology
25

Q. 38. What clinical test is being performed and its significance? Under
what circumstances it is necessary?
Image-Based Case Studies in ENT and Head & Neck Surgery

Q. 39. Describe the findings of this otoendoscopic image and give the
diagnosis. Identify the structure pointed by the arrow and its
significance.

26

Q. 40. This is an otoendoscopic image of a patient who had previous


history of recurrent ear discharge. Presently, he complained of
reduced hearing and blocked ear sensation.
a. Describe the findings and give the diagnosis.
b. What is the cause for his hearing loss?
c. How this patient should be managed further?
Q. 41. This is the endoscopic finding of the left ear of a child who
presented with acute otalgia and blocked ear sensation.
a. Describe the findings and give the diagnosis.

Otology
b. List the sequelae of this condition.
c. Outline the treatment.

27
Image-Based Case Studies in ENT and Head & Neck Surgery

28

Q. 42. This lady had ear piercing performed by a beautician 3 days prior
to consultation. Generally she felt unwell and the pinna feeling
hot.
a. Describe the findings and give the diagnosis.
b. List the potential complications that can occur.
c. Outline the treatment.
Q. 43. What procedure is being performed? Give its indications. List the
potential complications or adverse effects.

Otology
29
Answers
Image-Based Case Studies in ENT and Head & Neck Surgery

1. This is an endoscopic view of a normal ear canal and left tympanic


membrane (TM) (left because the handle of malleus is always directed
posteriorly). The ear canal has the characteristic anterior prominence
and is free from earwax. This is due to the outward bound epithelial
migration that starts from the umbo that radiates out right through the
ear canal. Thus, a normal ear is self-cleansing and there is no need
to use cotton bud. The TM is pearly translucent in appearance with
the handle of malleus positioned approximately 45 to the horizon
and the posterior malleolar ligament visible. The cone of light is at
the anterior inferior quadrant (around 8 oclock in the left ear and
4 oclock in the right ear) as this is the segment of TM that is 90 to
the ear canal and thus reflects the light of the otoscope or endoscope.
This position of the cone of light is distorted if the TM is abnormal
or retracted.
2. a. The normal translucent color of the TM is not present; instead it
looks yellowish due to the accumulation of fluid in the middle ear
space. The TM is retracted (handle of malleus is more acute than
usual 45) and the Prussaks space is retracted (a potential space
just above the lateral process of the malleus). This is consistent
with middle ear effusion (MEE) or serous otitis media.
b. The middle ear space is an air-filled cavity that extends posteriorly
to the mastoid antrum via the aditus and is connected to the
30 nasopharynx via the Eustachian tube (ET). The ET functions to
equalize pressure in the middle ear space and the atmospheric
pressure outside. Malfunctions occur most commonly due to
mechanical obstruction, which causes a gradual negative pressure
development in the middle ear that is eventually filled with fluid.
c. The causes of MEE include ET dysfunction (abnormal ET or
muscular movements of ET, e.g. cleft palate, craniofacial
abnormalities, mechanical obstruction, e.g. adenoids, tumors [e.g.
nasopharyngeal carcinoma (CA), very common cancer in Southeast
Asia], allergic rhinitis, and rhinosinusitis, etc.
3. a. This patient has undergone myringotomy and grommet (M&G)
insertion.
b. This procedure is indicated for persistent serous otitis media despite
conservative management, recurrent acute otitis media, marked
ET dysfunction and sometimes for intratympanic gentamicin
application in Menieres disease. This allows the fluid in the middle
ear space to be drained, and ventilation regained. As the grommet
stays in place for at least a few months, this hopefully will allow
the middle ear mucosa to heal and the ET recovers its function
and patency before it extrudes.
c. The myringotomy is performed at the anteroinferior quadrant as
this is the safest area. Posteroinferior quadrant should be avoided
at all cost as the ossicles (long process and lentiform process of
incus, stapes), and in some cases dehiscent facial nerve can be
injured at this quadrant. Manipulation of the ossicles at this area
can also dislocate the stapes footplate and result in a dead ear (total
loss of hearing and balance). Severe bleeding can occur in a high
jugular bulb, where the jugular bulb occupies the hypotympanum
or the lower limit of the middle ear space. A grommet is essentially
a tube, thus all complications related to tube applies; it can get
blocked, dislodged, and allow fluid from external ear to enter the
middle ear space predisposing to frequent middle ear infections.
This is commonly seen in children, who swim with their grommets
in situ.
4. a. The image shows a subtotal central perforation in quiescent stage
with the handle and neck of malleus left bare. The long process
of incus was absent and the lenticular process of stapes seen
surrounded by mucosal adhesions.
b. Rinnes test is expected to be negative and Webers test lateralized
to ipsilateral side indicating conductive hearing loss. As there was
ossicular discontinuity on the top of the large perforation, free field

Otology
voice test and audiometry would reveal severe conductive hearing
loss.
c. The options are hearing aid, surgery, or to leave it alone. Surgery
needs to be done at several stages including myringoplasty and
ossicular prosthesis insertion. The risk of incomplete closure, 31
secondary acquired cholesteatoma, and prosthesis-related
complications need to be considered. Care of the ear from
contaminants and treatment of intercurrent infections are important
along the way.
5. a. This is an abnormal and malformed external ear which is
called microtia (micro=small). It is a congenital lesion due to
malformation of the first and second branchial arches, which are
responsible for external ear formation. It is sometimes associated
with absent ear canal, which is called canal atresia as in this case,
or a narrow ear canal, e.g. canal stenosis. There is also an accessory
skin tag or vestigial auricle.
b. The management of this condition depends on whether the other
ear is normal. If the other ear is normal externally and hearing
wise, the patient can be advised to wait until he or she is old
enough to decide, if they want surgery for cosmetic reason. This
is because one functioning ear is enough for the patient to have
a normal hearing, speech and education. If the other ear is also
abnormal, then the child must have a bone conduction hearing aid
or even a bone anchored hearing aid fitted as soon as possible to
assist hearing. A computed tomography CT scan is performed at
the age of four to rule out any external ear cholesteatoma which
will require surgery. The middle ear and mastoid were analyzed
Image-Based Case Studies in ENT and Head & Neck Surgery

for degree of pneumatization and integrity of the ossicular chain,


and the normality of inner ear structures determined. A canal
reconstruction might be carried out in certain cases with canal
atresia. Reconstruction of the pinna can be performed later.
Nowadays, there is even bone anchored ear prosthesis that matches
the skin color.
6. a. There appears to be a pit or sinus just in front of the helix, which
is characteristic of a preauricular sinus. This is also a congenital
malformation related to incomplete closure of the branchial arches.
Although most commonly found in front of the helix, it can also
occur anywhere in the auricle.
b. They are usually asymptomatic. Sometimes, it can become
infected and exude purulent discharge. This can also result in
an abscess formation with a tender, fluctuant swelling which
can be accompanied by fever. The management depends on
the presentation. In asymptomatic patients, it can be left alone.
In infected cases, antibiotics may be required. Abscess needs
drainage and antibiotics, and subsequently requires an excision
to completely remove the sinus and its underlying tract in the
soft tissue. Inadequate excision of the tract can cause recurrent
infection. In rare cases, preauricular sinus is associated with
ossicular abnormalities known as Wildervanck syndrome.
7. a. The picture shows a keloid of the pinna. It can occur in anyone,
32 although more commonly in darker skin people. It usually occurs
after an injury or any trauma. The fundamental underlying pathology
is an exaggerated healing response which causes fibrosis out of the
boundaries of the actual injury. These swellings are firm and nontender.
b. Small lesions can be managed by multiple frequent injections
of steroids into the swellings while the larger ones will require
surgical excision, pressure bandage and subsequent steroid
injections (as justified). However, the keloid can still recur despite
the best possible management.
8. a. There appears to be a swelling at the external ear canal in the
bony segment (the medial aspect of the external ear). This is most
likely to be an exostosis. It is due to the bony outgrowth believed
to be caused by swimming in cold water. This can be confirmed
by palpating the swelling which will be bony hard in consistency.
The differential diagnosis includes tumors of the skin appendages,
e.g. seboma, etc.
b. In asymptomatic patients, it can be left alone. Surgical management
will be required in patients with obstructive symptoms, or in whom
where there is a difficulty to clean the ears with resulting earwax
cumulation and impaction. The exostosis can be drilled off but
there may be a risk of facial nerve palsy.
9. a. Examination revealed left Bells sign and the pinna appeared
erythematous with multiple blisters containing clear fluid.
b. Ramsay Hunt syndrome or herpes zoster oticus, which is varicella-
zoster virus infection involving the geniculate ganglion of facial
nerve in the temporal bone.
c. Laboratory test, e.g. total white count and erythrocyte sedimentation
rate can differentiate infective and inflammatory nature of the
lesions. Neurological studies has little role and not usually ordered
in typical presentation. Pain relief medications are given either
orally or by applying local anesthetic. Eye protection is necessary
to prevent corneal keratitis; however, this teenager showed positive
Bells sign, which had made the cornea completely concealed.
Corticosteroid in tapering doses and acyclovir are the main mode of
treatment. The latter has been shown to reduce pain and promote
resolution of symptoms if given within 48 hours of symptoms onset.
10. a. The otoscopic view reveals wax almost totally occluding the ear
canal.
b. Usually, the complaint of hearing loss in patients with earwax occurs
after swimming or digging the ears. Swimming or entry of water into
the ears causes the wax to expand and totally occlude the external
ear. This can be uncomfortable with a sense of ear obstruction,

Otology
which a child will complain of pain and not obstruction. In some
patients, they have dizziness and cases of persistent coughs
secondary to wax have also been reported. This is due to the vagus
nerve (Arnolds nerve) irritation as the nerve supply the innervation
to both (tympanic membrane) and the larynx. 33
c. In a child with impacted wax, removal either by syringing or
direct vision (under microscope) can be difficult due to the lack of
cooperation as the procedure can be painful. It is better to instill
wax softener, e.g. cerumol for few days to soften and dissolve the
wax before the procedure is undertaken. Syringing should not be
performed if the TM is suspected to be perforated.
11. a. There is a yellow round object at the opening of the ear canal
most likely a foreign body. Foreign body of the ear can be divided
into organic and non-organic. Organic foreign body can lead to
infection and need to be removed quickly. Insects, if alive can be
extremely painful and need to be killed with the use of any liquid.
Non-organic foreign body can be asymptomatic and left unnoticed
for some time.
b. The management of ear foreign body is by removal. A round
object would require a probe or a curved hook to be inserted
behind it, while the flat foreign bodies can be removed with the
forceps. Syringing can be used although if not used in the right
method can cause the foreign body to be pushed further inside.
Some cases of tightly impacted foreign body may require removal
under general anesthesia. Injuries to the TM or ossicles have been
reported in overzealous or unskillful attempt at the removal of
impacted foreign bodies.
Image-Based Case Studies in ENT and Head & Neck Surgery

12. This is a typical otoscopic view of otomycosis or fungal infection


of the ear. The black spores seen in the picture indicate Aspergillus
niger, while the white plague is often candidiasis. Together they
form a wet newspaper appearance. Otomycosis is predominantly
seen in humid tropical countries like Malaysia. Frequent, recurrent
otomycosis should be investigated for the possibility of diabetes
mellitus. The presentation can vary from itchiness and pain to a
sensation of blockage. Management involves a thorough cleaning of
the ear, typically under the microscope with a meticulous attention
to the inferior recess area. Antifungal ear drops, ointment, or powder
can then be used topically for at least 6 weeks.
13. This otoscopic picture reveals a central perforation (around 30%) with
purulent discharge noted around 9 oclock and the inferior recess area.
Thus, this indicates an active stage of chronic suppurative otitis media
(CSOM). The term tubotympanic (TT) indicates that the perforation is
safe that it does not have cholesteatoma and the possible complications
that can arise from it. However, TT is often not used and other term that
can be used to describe this condition includes CSOM (safe), CSOM
mucosal disease or just CSOM without cholesteatoma. In the presence
of active discharge, a thorough ear toilet is performed followed by
topical antibiotic drops or even oral antibiotic if justified. The patient
will be advised to keep his ear dry and subsequently, once it remains
dry for a period of at least 23 months, myringoplasty can be safely
34
performed. Preoperative audiogram is taken for record keeping. If
the ear continues to discharge despite antibiotics (after culture and
sensitivity), then cortical mastoidectomy needs to be considered. This
is to clear the infected mastoid air cells which would have served as a
reservoir for the persistent infection. (Note: Small perforation can close
by itself overtime provided the ear is dry and devoid of infection thus
not always need a repair).
14. a. There is an attic (area above the anterior and posterior malleolar
ligament or pars flaccida and scutum) pocket with white flaky
debris characteristic of cholesteatoma. The pars tensa appears
normal.
b. The discharge in these patients tends to be persistent, scanty, and
foul-smelling as compared to intermittent, copious, mucopurulent,
odorless discharge of CSOM (TT). Thus, the diagnosis is CSOM
[atticoantral (AA)]. Atticoantral indicates that the perforation
is unsafe; that it does have cholesteatoma and the possible
complications that can arise from it. However, AA is often not
used and other terms that can be used to describe this condition
include CSOM (unsafe), CSOM epithelial disease or just CSOM
with cholesteatoma.
c. The management of this condition is essential to prevent the
possible complications if it is allowed to expand; namely brain
abscess, labyrinthitis, facial palsy, subperiosteal abscess and
sigmoid sinus thrombosis. In a small cavity and a medically unfit
patient, regular cleaning will be adequate if all the boundaries of
the sac or cavity can be visualized and easily assessed with the
help of a microscope. If this is not possible, mastoid exploration
and exteriorization of cholesteatoma should be carried out. This
can be divided into an atticotomy, modified radical mastoidectomy
(MRM) or combined approach mastoidectomy depending on the
severity and intraoperative findings.
15. a. This is a left (the handle of malleus always points backward giving
a clue to if the ear is right or left) otoendoscopic view that reveals
a dark fluid collection in the middle ear space, probably blood.
The posterior malleolar ligament is also prominent with retraction
of the attic (Prussaks space) observed indicating the possibility
of a negative middle ear pressure. There appears to be a healed
perforation at around 7 oclock (anteroinferior aspect) suggesting
this patient might have had a grommet inserted before.
b. This finding is typically seen in hemotympanum. In this condition,
there is a collection of blood in the middle ear space usually

Otology
resulting from trauma. In some cases, it occurs secondary to severe
barotrauma when sudden changes of pressure occur while flying or
scuba diving in the presence of a nonfunctioning ET. This causes
an acute marked negative middle ear pressure with collection of
fluid or even blood in severe cases. Similar appearance can also 35
be found in cholesterol granuloma with MEE.
c. Hemotympanum is usually self-resolving and no surgical
intervention is necessary. Decongestants may be prescribed
and patient is advised to perform Valsalva maneuver if possible.
Antibiotic is not necessary. In a patient, without history of trauma,
middle ear fluid obtained upon myringotomy can be sent for
pathological examination if cholesterol granuloma is suspected
and, thereafter, the patient treated accordingly.
16. a. This picture reveals a post MRM cavity where mastoid, attic and
the external ear were operated and joined together to form a single
chamber. It is usually performed to exteriorize the cholesteatoma to
render the unsafe ear into safe ear. The middle ear cavity is usually
closed off by laying the TM over the medial aspect of middle ear space
since the scutum has been removed. This is important as an open
middle ear cavity with its respiratory mucosa (goblets cells and other
mucous secreting glands) will cause a discharging mastoid cavity. In
MRM, the facial ridge is lowered sufficiently to prevent sump effect
in the mastoid bowl.
b. Complications of this surgery include injury to the dura [with or
without cerebrospinal fluid (CSF) leak], labyrinth, facial nerve and
sigmoid sinus. There can also be a profound hearing loss that is
believed due to the drills used during the surgery.
Image-Based Case Studies in ENT and Head & Neck Surgery

c. Other procedures include atticotomy and combined approach


tympanoplasty. In atticotomy, disease that is limited to the attic is
removed by removing the scutum and exteriorizing the attic pocket.
This can be reconstructed with a cartilage grafting the same sitting
if the surgeon is confident that all the cholesteatoma has been
removed or perform later in the second sitting. Combined approach
tympanoplasty or canal wall up procedure is usually performed
in children, with well-pneumatized mastoid and in patients, who
complaint and understand that this is a two-stage procedure.
This procedure prevents a mastoid cavity and its complications
(discharging ear, regular ear toileting for life, vertigo and imbalance
in cold winds) and retains a normal external ear canal. It can be
technically more challenging as a posterior tympanotomy (boundary:
facial nerve, chorda tympani and buttress) is performed together
with the elevation of a tympanomeatal flap to remove the disease.
As recurrence rate is high, a second look is vital.
17. The right otoscopic view reveals an intact (tympanic membrane)
with a bulging mass behind it between 9 oclock and 12 oclock
position. The mass appears to be white in color and the presence of
blood vessels running over the mass continuous with the ear canal
confirms the presence of an intact TM. This appears to be a congenital
cholesteatoma, which refers to the persistence of embryonic epithelial
cell crest in the middle ear cleft. Congenital cholesteatoma can also
36 occur in petrous apex, cerebellopontine (CP) angle and mastoid.
Since the TM is intact, they do not present with the typical scanty,
purulent, persistent foul-smelling discharge that is the characteristic of
CSOM (AA). Thus, their main presenting symptom is usually hearing
loss or facial nerve palsy depending on the size and location of the
cholesteatoma. Management consists of a pure tone audiogram (PTA)
to assess and document the extent of hearing loss and a high resolution
computed tomographic (HRCT) scan that will be able to assess the
location, extent and other anatomical configurations that are important
in planning the gold standard of treatment: surgical Excision.
18. a. This otoendoscopy view of the left ear reveals an adhesive
otitis media. It usually occurs as a consequence of chronic ET
dysfunction that leads to chronic persistent negative middle ear
pressure. Eventually, the TM will be sucked in and draped or even
adherent to the medial aspect of the middle ear. The following
structures can be identified:
Handle of malleus
Lateral process of malleus
Umbo
Promontory
Round window niche
Long process of incus
Head of stapes
Stapedius tendon.
b. The hearing loss is minimal as the TM is draped and adherent
to the stapes forming a natural type III tympanoplasty (natural
myringostapediopexy). The presence of an intact TM prevents a
discharging ear.
c. Management of this condition needs to address two main factors.
The ET and the nasopharynx should be examined to rule out any
pathology and managed accordingly. As for the sensation of fullness,
M&G can be attempted if there is enough room to insert one.
With regards to the hearing loss, if it is significant and impairing
the quality of life of the patient, a hearing aid would probably be
the best solution. Surgical exploration and attempts to reventilate
the middle ear by elevating the TM is often unsuccessful with the
reformation of adhesions.
19. The otoendoscopy reveals an opaque TM with an anteroinferior
area atrophic (thinner) segment (estimated 20%), which indicates
a healed perforation. The location of the atrophic segment suggests
possibility of a previous grommet insertion site or probably a central
perforation complicating CSOM. The opaque TM indicates thickening

Otology
and scarring, and thus suggests frequent or longstanding middle ear
infection previously. Although the TM is intact, the atrophic segment
would not be able to vibrate as good as a normal TM and the energy
transmitted to the ossicles would be reduced. Audiogram will show
the severity of hearing loss. The management would be to advise the 37
use of hearing aid if the hearing loss is significant. There is no role
of surgery in this condition.
20. a. This is an endoscopic view of the left ear, where there appears
to be a white structure beneath the TM that appears to be thicker
than an abnormal TM. The attic region appears to be retracted and
could possibly have undergone surgery.
b. Based on the appearance, there are two possibilities. The white
structure could be the cartilage that was used (or even two pieces
of cartilage) that is usually used to reconstruct the attic after an
atticotomy, or the other possibility is that of incus interposition in
an ossiculoplasty or tympanoplasty.
21. a. The endoscopic view reveals a red, fleshy mass with surrounding
mucopus in the external ear canal. This appears to be an aural
polyp. It can occur due to infection, cholesteatoma, malignant
otitis externa, or even malignancy.
b. In this patient, ear swab was taken for culture and sensitivity,
and thereafter, a thorough aural toilet and examination under
microscope was performed. It is important to probe all around
the polyp to see where the stalk or where the polyp is coming
from. If it is arising from middle ear, the probe will be able to
go all around the polyp. A trial of antibiotic can be given and
subsequently polypectomy carried out using a snare. If it is arising
Image-Based Case Studies in ENT and Head & Neck Surgery

from the posterosuperior aspect, then there is a strong possibility of


cholesteatoma and a mastoid exploration after CT scan would be
required. Biopsy of the polyp is taken to rule out any malignancy.
22. a. There appears to be a red, granular mass in the introitus of the left
ear canal. Considering the age of the patient (white hair!) and the
presenting symptom of bleeding, a malignancy, e.g. CA of the ear
would be the provisional diagnosis. Differential diagnosis would
include ear polypor granulation. The presence of facial nerve palsy
would usually support the diagnosis of CA ear.
b. Management would consist of a thorough ear toilet and examination
under microscope if possible, biopsy and a high definition CT scan.
If biopsy confirms malignancy, CT scan should be assessed to look
for the involvement of mastoid, middle and inner ear, and the
extension beyond the temporal bone including neck metastasis,
parotid gland and temporomandibular joint. Surgical management
would depend on the extent of local involvement ranging from
a sleeve resection for an ear canal CA, a lateral temporal bone
resection if the inner ear is not involved (removal of all bone and
tissue lateral to facial nerve and closure of ear canal in a blind sac
fashion), a subtotal or total temporal bone resection if inner ear
is involved, and a combined neck dissection with parotidectomy
if these are involved. Radiotherapy is given postoperatively. The
prognosis is quite poor for extensive lesions.
38 23. a. This is an axial CT scan of the skull base and temporal bone (bony
window).
b. (1) Mastoid air cells (well-pneumatized), (2) Jugular bulb,
(3) Pneumatization of zygoma, (4) Ossicles, (5) Cochlea,
(6) Sphenoid sinuses, (7) Internal carotid artery [horizontal (right)
and vertical (left) portion in petrous apex], (8) Sigmoid sinus and
(9) External auditory canal.
24. a. This is a coronal view of magnetic resonance imaging (MRI) scan
most probably with gadolinium. It reveals a lesion at the CP angle
most likely due to a vestibular schwannoma (VS) [also commonly
known as acoustic neuroma].
b. The differential diagnoses of CP angle mass include meningioma,
aneurysm, cholesteatoma and cholesterol granuloma. However,
the presence of an extension of the tumor into the internal auditory
meatus (IAM) is almost pathognomonic for VS (as seen in this
image). Meningiomas are usually more broad based, does not
extend into the IAM and frequently has a dural tail.
c. The clinical presentation of a VS ranges from purely otological
symptoms in early stage to neurological in more advanced stage. In
early cases, where the tumor is localized in the IAM (usually < 2 cm),
patients present with asymmetrical hearing loss or occasionally
sudden hearing loss. Occasionally, tinnitus and imbalance can be
the presenting symptoms. In larger lesion (> 2 cm), cranial nerve
symptoms (fifth, seventh and ninth, tenth) can be present. The
earliest being loss of corneal reflex and paresthesia of the cheek
(trigeminal nerve). Cerebellar compression causes imbalance and
brainstem compression can be eventually fatal. Large lesions can
cause hydrocephalus. They rarely present with facial nerve palsy.
d. Management takes into account multiple factors including
age, hearing level, general medical condition and the patients
preference of treatment. This ranges from observation with yearly
MRI for small lesion (1.5 cm) (Note: acoustic neuroma is a slow
growing tumor), radiosurgery for lesions below 22.5 cm in elderly
patients, and surgical removal in patients with large tumors that
indents the brainstem and cerebellum.
25. This appears to be an endoscopic view of the right TM. The pars
flaccida (attic) is retracted, specifically at the area of Prussaks space
(above the lateral process of the handle of malleus, scutum and the
neck of malleus). There is an evidence of previous M&G insertion
with the presence of a healed perforation (TM thinner than normal)
at around 4 oclock. The yellowish discoloration over the healed
perforation can be due to adhesion to the promontory or the presence

Otology
of serous otitis media (glue ear). This is typically seen in longstanding
ET obstruction commonly from childhood itself. These patients
usually present with sensation of ear fullness and blockage, inability
to equalize the ears, and in some cases hearing loss.
39
26. Brainstem evoked response audiometry (BERA). It is an objective
audiological testing, wherein sound stimulus generates electrical
responses in the cochlea (inner ear) and in the brainstem. This
response travels along the auditory pathway, resulting in waveforms,
identifiable to the site of origin by the amplitude and latency. The
advantages of BERA include noninvasive, reliability on repetitive
testing, and not affected by sedatives or drugs. Thus, it is most useful
in newborn and neonates. In adult, it is mostly ordered for acoustic
neuroma, to differentiate cochlear and retrocochlear lesion, and in
malingering patients.
27. a. Conductive hearing loss in the presence of normal TM suggests a
middle ear pathology. These may range from tympanosclerosis,
ossicular discontinuity, adhesion, or even ossicular fusion. This
patient is very likely to have otosclerosis, a condition whereby
there is a new bone formation at or about the stapes footplate thus
impeding the transmission of sound to the oval window. The lesion
is more commonly found in females and hormonal changes during
pregnancy would have accelerated the disease process.
b. Figure A shows a complete stapes structure, i.e. the footplate,
anterior and posterior crus, and its head. Figure B shows a whitish
material encircling and anchored at the lower part of the long
Image-Based Case Studies in ENT and Head & Neck Surgery

process of incus.
c. These figures gave a clue to stapedectomy with tympanoplasty
performed. Ossicular reconstruction can be done with various
commercialized materials, e.g. titanium piston, or synthetic material
such as plastipore (likely in this case since it is white in color). This
would restore the sound transmission to the round window enabling
closure of air-bone gap.
28. a. Figure A showed mastoid and postauricular skin bruise, the typical
Battles sign while Figure B showed a bluish spherical rim in the
middle ear (an indicator of resolving hemotympanum) and purplish
bruise along and around the handle of malleus with no obvious
clue of ear canal injury. These signs are highly suggestive of skull
base fractures involving the squamous temporal bone (usually
transverse fracture if TM is intact). In contrast, the presence of
these findings in infant and young children would be suspicious
of nonaccidental injury or child abuse.
b. Cranial nerves examination in particular, the facial and auditory
nerves, to document paralysis and hearing loss, respectively.
Presence of conductive hearing loss without immediate facial
palsy, sensorineural hearing loss (SNHL), or vestibular symptoms
are in favor of longitudinal fractures rather than transverse fractures
of the temporal bone.
c. In this patient, there is no facial palsy or nystagmus noted and the
40 hearing loss is mild and conductive in nature, thus a conservative
management is chosen.
29. This is typical of a subperiosteal abscess that is seen as a complication
of acute mastoiditis. This can be an emergency condition. This
child needs admission, intravenous antibiotics, CT scan to rule
out intracranial complication (upto 30% of the children can have
asymptomatic intracranial complication) with the definitive treatment
of surgical exploration of the mastoid. The postauricular incision
should be higher in children due to the incomplete elongation of the
mastoid tip as the facial nerve can be injured in a normally placed
postauricular incision.
30. This is a PTA of a patient performed in October 2001. It reveals
a dip at 4 kHz of upto 6070 decibel (dB) with normal hearing
threshold in all other frequencies. This is typically seen in noise-
induced hearing loss. The dip at 4 kHz is believed to be due to the
fact that the promontory (corresponds to 4 kHz) is the most sensitive
part of the cochlea, therefore, vulnerable to noise insults. It initially
presents as a temporary threshold shift (in early stage) that recovers
to normal hearing threshold. Permanent damage leads to a similar
audiogram as above. This is usually due to the exposure of noise
above 90 dB, 8 hours a day, 5 days a week. If this patient is working
in a noisy environment, it is vital that he is referred to occupational
health authorities and provided with appropriate ear protection while
working.
31. a. Otoscopic examination reveals a reddish mass in the inferior part
of the left middle ear behind the intact TM. This appears to be
arising from the floor of the middle ear, extending to the umbo.
b. This MRI shows a hyperintense or hypervascular mass at the left
middle ear space. It seems to be localized in this area without any
extension into mastoid air cells.
c. The possible diagnosis includes glomus tympanicum, glomus
jugulare and possibly high jugular bulb.
d. The main treatment is excision of the lesion. She needs a preoperative
angiogram and embolization to reduce the risk of bleeding from the
lesion intraoperatively. The other option is conservative, especially if
the lesion is not growing and the patient is not fit for surgery.
32. a. This HRCT scan shows a longitudinal fracture of right temporal
bone from ear canal toward the petrous apex, traversing the middle
ear space with soft tissue density at the right mastoid cavity, most
likely blood accumulation after temporal bone fracture.
b. The diagnosis is a longitudinal fracture of right temporal bone with
hematoma.

Otology
c. The possible complications from this lesion include conductive
hearing loss, facial nerve palsy, vertigo and CSF leak.
d. The management is usually conservative and depends on the
complications of the fracture. Delayed onset of facial nerve paralysis is
treated conservatively, while immediate onset of total facial paralysis 41
may require surgery in the form of decompression, reanastomosis of
cut ends or cable graft. Hearing test is always needed during follow-up
to assess any hearing loss. Neurotological examination and assessment
is important as labyrinth concussion may occur.
33. a. This child has had right cochlear implant performed.
b. The indications of cochlear implants include severe to profound
bilateral SNHL (unaided threshold >90 dB) with little or no
benefit from 3 months to 4 months trial of hearing aid. This may
be performed for congenital hearing loss in children ideally before
the age of four or in postlingual deafness in adults or older children.
Children with congenital hearing loss detected after the age of four
may not have good results.
c. The necessary preoperative evaluations include trial of hearing aid
for 36 months, no medical contraindication to undergo surgery,
good family support, preoperative HRCT to assess the anatomy of
the inner ear, otoscopic examination to rule out any middle ear
infection and assessment by audiologist and speech therapist.
d. The child needs to be followed up with the experienced audiologist
and speech therapist for cochlear implant rehabilitation. This is
probably the most crucial part of the program.
34. a. There appears to be a red and angry looking swelling of superior
part of the left pinna.
Image-Based Case Studies in ENT and Head & Neck Surgery

b. This is most likely a left pinna abscess.


c. The patient needs to be admitted for hydration, intravenous
antibiotics and later incision and drainage with anesthesia.
Corrugated drain needs to be inserted to drain the pus for the next
few days. Later, she needs to be followed up with regular daily
dressing till the wound is clean and healed. She would require
oral antibiotics upon discharge for the perichondritis.
d. The potential complications that can occur includes septicemia,
perichondritis and cauliflower ear.
35. a. This otoendoscopic examination reveals two central perforations of
left TM, one at anterior quadrant and the other at posterior quadrant
with ragged perforation edges and there is blood clot noted at
the superior edge of the posterior perforation. The remaining TM
appears hyperemic.
b. Hearing test and audiometry are vital to assess the extent of the
hearing loss mainly for medical legal documentation as well as
comparison during follow-up.
c. The chances of traumatic TM to recover spontaneously are high
as long as there is no superimposed infection. However, the
hearing and balance will have to be assessed as well as the inner
ear function can be compromised in trauma.
36. a. There appears an area of periantral sclerosis, which is typically
seen in cholesteatoma.
42 b. This should be diagnosed as cholesteatoma, until a microscopic
examination reveals otherwise.
c. Cholesteatoma can erode bone and cause intracranial (meningitis,
intracranial abscesses) and extracranial (facial nerve palsy, sigmoid
sinus thrombosis, labyrinthitis and subperiosteal abscesses)
complications.
d. This child will require surgical exploration of the attic and mastoid
cavity; most likely a MRM.
37. a. The step shown in this image is harvesting of the temporalis fascia
graft for myringoplasty.
b. The role of a graft in myringoplasty is as a scaffold for the epithelium
of the perforation edges to epithelialize and close the perforation.
c. Other materials that can be used for similar purpose include
perichondrium, cartilage, periosteum and fat tissue.
38. The clinical test that is being performed is masking Rinne tuning fork
test. The significance of this test is to mask the non-tested ear with
Baranys noise box to decrease the crossover effect from the normal
non-tested ear. This test is necessary in severe unilateral SNHL (the
right ear in this instance).
39. This is an otoendoscopic image of the left eardrum which shows
Grade IV pars tensa retraction with the eardrum adhered to medial
wall of the middle ear cavity. The pointed bluish structure is most
likely a high jugular bulb which occasionally be seen as an anomaly
in this region.
40. a. The otoendoscopic image shows two atrophic central TM
retraction. There appears to be straw-colored MEE. This is most
likely healed doubled perforations from previous CSOM. Absence
of scar made history of previous myringoplasty unlikely.
b. This patient most likely has right conductive hearing loss due to
persistent glue ear.
c. Structural and functional cause that gives rise to ET dysfunction
need to be ruled out and treated accordingly. Grommet insertion
is an alternative measure but the risk of nonclosure should be
borne in mind in this paper-thin atrophic healed perforation.
41. a. The otoendoscopic finding is bulging, and hyperemia right TM
with loss of cone of light. This is presuppurative stage of acute
suppurative otitis media.
b. Sequelae of this condition are TM perforation, nonsuppurative
MEE, adhesion, tympanosclerosis, erosion of ossicular chain, and
hearing loss (conductive and SNHL).
c. The principles of further management are:
Antibacterial therapy

Otology
Decongestant nasal drops
Analgesics and antipyretics
Myringotomy to be performed when:
The eardrum is bulging and there is severe acute pain
Incomplete resolution despite antibiotics with the eardrum 43
remains full with persistent conductive hearing loss
Persistent effusion beyond 12 weeks.
42. a. These photos show red, swollen right pinna. This is acute right
pinna perichondritis.
b. The potential complications that can occur are pinna abscess,
sepsis and cauliflower ear.
c. The treatment consists of parenteral antibiotics, analgesic and
antipyretics. If abscess develops, incision and drainage needs to
be performed.
43. The procedure that is being performed is ear syringing.
The indications of ear syringing are impacted earwax and removal
of non-organic foreign body in the ear.
The potential complications include TM perforation, dislocation of
ossicular chain, conductive hearing loss, SNHL, acute dizziness, and
facial nerve palsy.
2 rhinology

Questions

B
Q. 44. These are endoscopic images of the right nasal cavity.
a. What does it show?
b. What is the clinical implication?
c. How would this patient be managed subsequently?
Q. 45. This patient presented with painful, tender nose.
a. What is the diagnosis?

Rhinology
b. What are the possible complications?
c. Outline this patients management.

45

Q. 46. This patient presented with a longstanding history of nasal


obstruction bilaterally.
a. What does this examination reveal?
b. How should this patient be managed further?
Image-Based Case Studies in ENT and Head & Neck Surgery

Q. 47. This is a view obtained during an endoscopic nasal examination


of the left nasal cavity.
a. What is the abnormality?
b. What are the possible clinical implications?
c. How would this lesion be managed?

46

Q. 48. This patient presented with recurrent bleeding from the left nostril.
a. What is the diagnosis?
b. How would this patient be managed further?
Q. 49. This patient was seen in the clinic with nasal symptoms.
a. What are the diagnoses and its possible etiologies?

Rhinology
b. What are the clinical manifestations?
c. How would this condition be managed further?

47

Q. 50. The above patient was seen in an allergy clinic.


a. What is the sign demonstrated?
b. What is the cause?
Image-Based Case Studies in ENT and Head & Neck Surgery

Q. 51. This is the nasal endoscopic view of a patient who presented


with nasal obstruction.
a. What does this image show?
b. What are the possible etiologies?
c. What further treatment would be necessary?

48
A

Rhinology
49

B
Q. 52. This is the clinical finding in a patient who presented with fever
and acute facial pain.
a. What is the diagnosis?
b. Outline the subsequent treatment.
Image-Based Case Studies in ENT and Head & Neck Surgery

50

B
Q. 53. The above patient presented with recurrent acute sinusitis.
Endoscopic finding and computed tomography scan of the
paranasal sinuses are enclosed.
a. What is the diagnosis?
b. Outline the management of this patient.
A

Rhinology
51

B
Q. 54. This patient presented with nasal bleeding and obstruction. What
is the diagnosis and its management?
Image-Based Case Studies in ENT and Head & Neck Surgery

Q. 55. This patient presented with the loss of smell. What is the diagnosis
and its subsequent management?

52
A

Rhinology
53

B
Q. 56. The above child presented with persistent unilateral nasal
discharge. How would this child be managed further?
Image-Based Case Studies in ENT and Head & Neck Surgery

54

B
Q. 57. The above patient presented with nasal discharge. Nasal
endoscopy followed by suction was performed and the findings
were as shown respectively.
What is the likely diagnosis and subsequent management?
A

Rhinology
55

B
Q. 58. The above patient presented with ulceration of the hard palate.
Computed tomography scan is enclosed.
What is the diagnosis and subsequent management?
Image-Based Case Studies in ENT and Head & Neck Surgery

56

B
Q. 59. A Chinese gentleman presented with occasional epistaxis and
reduced hearing.
a. Describe the findings on endoscopic evaluation.
b. What is the most likely diagnosis and its further management?
A

Rhinology
57

B
Q. 60. This patient presented with clear nasal drips after an endoscopic
nasal procedure.
What is the diagnosis and subsequent management?
Image-Based Case Studies in ENT and Head & Neck Surgery

58

B
Q. 61. This is a picture of a patient, who developed right eye swelling
and reduced eyesight 1 week after he suffered an upper respiratory
tract infection.
What is the diagnosis and further management?
Q. 62. This is an endoscopic view of the nasopharynx in a child.
What is seen and how should this case proceed?

Rhinology
59

Q. 63. This picture was taken during an endoscopic evaluation of the


nasal cavity.
a. Describe the findings.
b. What is the diagnosis and its clinical implications?
Image-Based Case Studies in ENT and Head & Neck Surgery

Q. 64. This patient had an operation done to relieve his left nasal
obstruction. The endoscopic view is as shown.
a. Describe the findings.
b. What procedure would probably have been done?
c. What precautions are to be considered in this surgery?

60
A

Rhinology
61

B
Q. 65. A 13-year-old male presented with progressive left nasal
obstruction and occasional epistaxis. There were no other ENT
complaints of significance.
a. What test is being performed on the patient?
b. Describe the endoscopic findings.
c. What is the most likely diagnosis and its subsequent management?
Image-Based Case Studies in ENT and Head & Neck Surgery

62

B
Q. 66. This middle-aged lady had a surgery done to relieve her right
watery eye. On follow-up 3 weeks later, her eye was re-examined
and nasal endoscopy was performed.
a. Comment on the images shown.
b. What is the subsequent management?
A

Rhinology
63

B
Q. 67. A young gentleman presented with a dull headache for few
months duration, which has become persistent at the time of
consultation. Nasal endoscopy was performed followed by
imaging study.
a. Describe the imaging study findings.
b. What are the possible diagnoses?
c. How would this patient be managed further?
Image-Based Case Studies in ENT and Head & Neck Surgery

64

B
Q. 68. A lady presented with persistent left nasal obstruction for many
years, but recently sought consultation due to on-and-off blood-
stained nasal discharge, which sometimes becomes smelly.
Endoscopic findings were performed followed by imaging study.
a. Describe the nasal findings and correlate it with the radiology
image.
b. Give the diagnosis.
c. What is the etiology?
d. How would this patient be treated further?
Q. 69. This child was noted to have a slowly enlarging discolored lesion
involving her nose. There were no other similar lesions elsewhere
and the rest of the ENT examinations were unremarkable.
a. Describe the findings.

Rhinology
b. What is the diagnosis?
c. Outline treatment modalities available for this lesion.

65

Q. 70. This elderly patient presented with a slowly enlarging swelling


involving his nose. There were occasional itchiness but no
obvious pain noted.
a. Describe the findings.
b. Give provisional and differential diagnoses.
c. Outline further management of this patient.
Image-Based Case Studies in ENT and Head & Neck Surgery

Q. 71. An adult male patient allegedly fell from his bicycle and hit the
nose against gravel. There were some epistaxis noted and the
bridge of the nose became swollen.
a. Describe the X-ray findings.
b. How would it be fixed?
c. What other lesions should not be missed?

66

Q. 72. This is an endoscopic image of a patient who had an endoscopic


sinus surgery previously. Currently he is well and has no nasal
symptom.
a. Describe the findings.
b. What is the diagnosis?
c. How should he be treated?
A

Rhinology
67

B
Q. 73. A middle-aged female presented with nasal discoloration and right
side nasal blockage for a month duration. She had surgery done
previously to make her nose look better. There were no obvious
pain, fever, or other nasal symptoms and she is a nondiabetic.
a. Describe the findings of the external nose and right nasal
endoscopy.
b. What are the possibilities?
c. How would this patient be treated further?
Image-Based Case Studies in ENT and Head & Neck Surgery

Q. 74. This lady was diagnosed to have nasopharyngeal carcinoma and


treated 10 years earlier. She presented now with blurring of vision
and examination revealed ophthalmoplegia with visual acuity
result of perception to light.
a. Describe the findings.
b. What is the diagnosis?
c. Give the prognosis.
68
A

Rhinology
69

Q. 75. These are the nasal endoscopic images of a patient who presented
with unilateral nasal blockage.
a. Describe the findings and give the diagnosis.
b. How would this patient be managed further?
Image-Based Case Studies in ENT and Head & Neck Surgery

Q. 76. Describe this intraoperative image and discuss the proposed


surgery.

70
Answers
44. a. This is an endoscopic view of the right nasal cavity. The inferior
turbinate is seen inferiorly and the middle turbinate appears
double with some crustings on its surface. Thus, this could be a
bifid turbinate or the lateral structure could be a large uncinate
process.
b. The clinical implication is due to the anatomical abnormality that
may obstruct the drainage and ventilation of the middle meatus.
This will cause disruption of the mucociliary clearance of the
sinuses draining into the osteomeatal complex (OMC) and lead to
chronic rhinosinusitis. These patients can present with recurrent
acute sinusitis, facial heaviness and congestion, postnasal drip
and nasal obstruction. Nasal congestion can also lead to mouth
breathing and snoring.
c. Management of most anatomical abnormalities usually require
surgical intervention if symptomatic as they do not usually resolve
with medical therapy. In this case, surgery to clear the middle
meatus to establish normal ventilation and drainage of the sinuses

Rhinology
will resolve the problems as mentioned.
45. a. This patient is most likely having nasal skin infection. However,
underlying vestibulitis should be looked for. In this condition,
infection occurs in the nasal vestibule area (the hair-bearing skin
of the nasal cavity upto the nasal valve) usually due to minor
trauma of the underlying nasal mucosa usually from nose picking. 71
The common organisms implicated are usually Staphylococcus
aureus and Streptococcus pyogenes. The clinical features include
indurated skin, swelling, and tenderness of nasal alar or the tip
of the nose, with or without sticky nasal discharge, which can be
purulent.
b. Severe cases can lead to cellulitis of the surrounding soft tissue
which can have serious consequences as this is the dangerous zone
of the face. Infection of this area can possibly cause retrograde
thrombophlebitis via the ophthalmic vein into the pterygoid
plexus and the cavernous sinus. Underlying immunocompromised
conditions should be ruled out.
c. Management consists of antibiotics (intravenous in cases of cellulitis)
with appropriate analgesics. In chronic cases or recurrent infections,
fusidic acid ointment may be applied to the nasal vestibule.
46. a. The nasal examination reveals a deviated nasal septum (DNS)
to the left with a compensatory hypertrophy of the right inferior
turbinate. The nasal obstruction on the left is further compounded
by the fact that the DNS occurs at the nasal valve region. This area,
i.e. nasal valve is the narrowest part of the entire upper airway and
contributes to at least 50% of the airway resistance.
b. The initial management of this patient consists of a thorough history
and clinical examination with endoscopic evaluation to exclude
Image-Based Case Studies in ENT and Head & Neck Surgery

other causes of nasal obstruction. Once the diagnosis is confirmed


to be DNS, a septoplasty is warranted to correct the DNS with
some form of right turbinate reduction procedure or surgery. It
must be explained to the patient that the septal correction would
increase the left airway but the right airway could be reduced as
compared to preoperatively since the septum will return to the
normal place.
47. a. There appears to be a spur arising from the left side of the nasal
septum. It is usually a congenital malformation but can also result
from trauma. Commonly, the spur is formed at the junction of the
bony maxillary crest and the nasal septal cartilage.
b. Patient is usually asymptomatic if it is small, but can experience
nasal obstruction if it is large. In some patients, the spur impinges
on the middle turbinate causing shooting pain along the nasal
bridge. This is known as Sluders neuralgia, where it is believed
to be due to the compression of the ethmoidal nerves.
c. Management of this condition depends on the symptom. It can
be left alone if asymptomatic. Persistent symptoms would be an
indication of surgical excision of septal spur.
48. a. This is the Littles area of the nose, the area of the anterior portion of
the nasal septum most vulnerable to injury particularly during nose
picking. It is very vascular supplied by four arteries and commonly
bleeds particularly in children. In children, it is believed to be due
72 to the changes in temperature that triggers the bleeding from this
area. Prominent subepithelial vessel branches were seen in this
patient that would have contributed to the epistaxis events.
b. Management is to complete an endoscopic nasal examination
if bleeding persists or becoming recurrent to exclude any other
pathology. The prominent vessels can be cauterized. If the septum
is deviated, septoplasty could help in addressing the problem.
49. a. This is an endoscopic view of the right nasal cavity that reveals a
septal perforation. There appears to be some amount of discharge
around it as well. The possible etiologies include postsurgery
(iatrogenic), chronic infection (leprosy, tuberculosis, granulomatosis,
etc.), or even malignancy. In this case, the perforation is small and
the edges appear quite smooth without obvious granulations that
are commonly seen in chronic infection or malignancy. Thus, this
is most probably an iatrogenic complication.
b. The clinical features range from asymptomatic to whistling sound,
bleeding and crustings.
c. The management is directed by the symptom; asymptomatic
cases can be left alone. Symptomatic perforations, after excluding
infection or malignancy (by biopsy of granulation edges) should
be managed with closure by surgery or prosthesis.
50. a. The above patient has a pigmented horizontal crease along dorsum
of the nose as well as along either nasolabial folds. This finding
is typically found in a patient, who demonstrates the act called
allergic salute.
b. It is due to allergic rhinitis, where frequent wiping or rubbing
(usually upward) of the nose that is associated with rhinitis
causes crease formation along its dorsum with variable degree of
pigmentation. The management is to treat the underlying cause
as most patients do not even notice the crease.
51. a. The image is of the right nasal cavity. It shows a mild deviation
of the upper nasal septum to the right with inferior turbinate
hypertrophy that is compromising the right nasal airway.
b. The etiology of the inferior turbinate hypertrophy is multifactorial
ranging from compensatory enlargement secondary to nasal
septum deviation to allergy, vasomotor rhinitis, hormonal changes
and chronic infection.
c. The management consists of determining the initial causative
factor and treating the underlying cause if possible. Following
which, a course of nasal corticosteroids and/or antihistamine with
or without decongestants can be used. If it persisted despite this

Rhinology
treatment, surgical intervention can be considered, ranging from
surface cautery and submucous diathermy to the more invasive
turbinoplasty or even turbinectomy.
52. a. Image appears to be an endoscopic view of the left nasal cavity
that reveals a mucopurulent discharge from the left middle meatus.
This is the characteristic of acute maxillary sinusitis. The presence 73
of pus discharging from the middle meatus is a good indicator as
it suggests the maxilla is draining and the ostium is patent. Acute
maxillary sinusitis, that is not draining tends to present with acute
facial pain and in severe cases can cause complications, e.g. to
the orbit. The sinus X-ray shows opacity of the left maxilla that is
consistent with sinusitis. It is important to note that a unilateral
recurrent maxillary sinusitis could be due to dental infections as
the upper molars are embedded into the floor of the maxilla.
b. The management consists of antibiotics, decongestants and
analgesics in the initial stages. Recurrent episodes would indicate
an underlying dental pathology or chronic rhinosinusitis, which
should be treated accordingly.
53. a. The endoscopic view of the right nasal cavity reveals a large
middle turbinate and the computed tomography (CT) scan confirms
the clinical impression of concha bullosa. This is defined as a
pneumatized middle turbinate or simply the presence of air cell
within the middle concha. It is probably one of the most common
anatomical anomalies seen in the nasal cavity. Concha bullosa
may remain asymptomatic and only be found incidentally on nasal
endoscopy. However, it can block the OMC, which would lead to
reduced ventilation and drainage of the sinuses; thus, predispose
to recurrent acute sinusitis or chronic rhinosinusitis.
Image-Based Case Studies in ENT and Head & Neck Surgery

b. The treatment is usually surgical as most anatomical abnormalities


do not respond to medical therapy. The CT scan shows quite clearly
the large pneumatized middle turbinate and DNS to the left. Thus,
the pneumatization could be a compensatory mechanism. The
OMC appears to be compromised in the CT scan, which would
have predisposed this patient to recurrent sinusitis.
54. The right nasal cavity appears to be filled with a fleshy vascular mass
medial to the inferior turbinate. The CT scan reveals opacity of the
right nasal cavity, OMC, and ethmoid sinuses with mucosal thickening
of the maxillary sinuses. There is no bone erosion or destruction
noted, thus this tumor is probably benign. Management includes a
thorough history, clinical examination with nasal endoscope and a
biopsy to confirm the histological diagnosis. A unilateral nasal mass
without bony destruction is most likely to be an inverted papilloma
or antrochoanal polyp. The absence of opacity in the maxillary sinus
in the CT scan excludes antrochoanal polyp.
55. There appears to be multiple smooth, glistening swellings in the right
nasal cavity, which are typical of nasal polyps. They usually present
with nasal obstruction and loss of smell. Facial pain and discomfort is
not common, which helps to differentiate this condition from chronic
rhinosinusitis. If they are bilateral, they are ethmoidal polyps, while
antrochoanal polyps are unilateral in nature. The management consists
of an initial biopsy to confirm the diagnosis and followed by nasal
74 corticosteroid spray or even oral steroids in severe cases. In cases where
the obstruction is complete, surgical modality may be employed.
56. There appears to be a brownish mass in the right nostril, which turned
out to be a fruit seed (foreign body). This diagnosis must always be
considered in a child with persistent unilateral nasal discharge. The
foreign body can be removed in the clinic with the child wrapped or
held tightly by his/her parent with the use of right instrument. Failure
of which, removal under general anesthesia (GA) would be required.
57. The nasal cavity appears to be filled with slate-brown material covered
with purulent discharge. This finding is usually seen in patient with
fungal sinusitis and would require a thorough nasal endoscopy,
removal of infected material, and mucosal biopsy. Secretion should
be sent to look for fungal hyphae. Once the diagnosis is confirmed, CT
scan of the paranasal sinuses (PNS) is ordered to assess the extent of the
disease. Endoscopy-assisted surgical debridement can be performed
and in cases of invasive fungal sinusitis in immunocompromised
patient, systemic antifungal may be required.
58. The ulceration appears located at the posterior aspect of hard palate
and its junction with the soft palate. It has an exophytic edge with
marked areas of granulations. The dorsal surface of the tongue is
covered with white material, probably an oral thrush. The CT scan
showed complete opacification of the left maxillary cavity with
erosions involving its floor and the lateral nasal wall. The inferior
turbinate and part of the nasal septum were destroyed as well. Bony
erosion and destruction usually indicate a malignancy. In this case,
the diagnosis is most likely carcinoma (CA) of maxilla, which would
have started from its floor. Management consists of confirming the
diagnosis with histopathological examination (HPE) and investigation
to exclude distant metastasis. Total maxillectomy, followed by
radiotherapy or chemotherapy as advised by the oncologist would
be the recommended scheme of treatment.
59. a. The nasendoscopic view of the nasopharynx reveals an
ulcerofungating mass involving the fossa of Rosenmller (FOR) and
the posterior wall of nasopharynx. The right otoendoscopic view
reveals a retracted tympanic membrane with yellowish fluid in the
middle ear (serous otitis media, middle ear effusion, glue ear).
b. These are typical findings of nasopharyngeal carcinoma, which
is a common malignancy especially among the Chinese. It is
believed to be due to Epstein-Barr virus, although the triggering
factor is still unknown. The obstructive effect of the tumor on the
Eustachian tube (ET) causes a negative middle ear pressure and

Rhinology
accumulation of fluid in the middle ear space. Once the diagnosis
is confirmed histologically (to exclude lymphoma especially
in WHO type III), imaging modalities (computed tomography,
magnetic resonance imaging, positron emission tomography
scan, bone scan, ultrasound of liver) are used to determine its
local extent, presence of distant metastasis, and help in staging 75
of the disease. Treatment options include radiotherapy with or
without chemotherapy depending on the stage. Myringotomy and
grommet insertion will alleviate hearing loss while the tumor is
being treated.
60. This is a bone window view of coronal cut CT scan of the PNS. There
is a defect of the right fovea ethmoidalis with herniation of intracranial
content into the nasal cavity. A clear drip after an endoscopic sinus
surgery should always be presumed a cerebrospinal fluid (CSF)
leak. Initial management includes confirming the fluid as CSF (beta-
transferrin test) and investigation to locate the site of leak. If the defect
is large, CT scan would be able to demonstrate the exact location.
However, smaller defect may be missed. Sometimes, a fluorescein
dye is used intrathecally to identify the leak. If the leak persists
(most small leaks heal spontaneously), an endoscopic repair may be
carried out. The choice of graft material for repair ranges from septal
mucoperichondrial graft to fascia lata. In some cases, lumbar drain
is inserted by some surgeons.
61. The picture shows an orbital complication of acute sinusitis. The right
eye appears swollen with its conjunctiva suffused and edematous
which also seem to be pushed inferiorly. This is usually due to
subperiosteal cellulitis, which complicates to abscess formation as
a result of extension of pus from the ethmoid sinusitis breaching
Image-Based Case Studies in ENT and Head & Neck Surgery

the lamina papyracea. The CT scan illustrates this well on the right
side, where there is opacity of the right ethmoid with collection of
pus between the medial wall of the orbit and the eyeball itself. This
causes the orbit to be compressed and may eventually result in vision
loss; thus this is an emergency. The patient needs to be admitted and
started on intravenous antibiotics. An ophthalmology consultation
should be obtained and if there is vision reduction or the presence of
sub-perichondrial abscess in the CT scan, the pus should be drained
via open Lynch-Howarth incision or endoscopically according to the
surgeons experience.
62. The image shows a moderate size adenoid tissue along the roof and
posterior wall of nasopharynx. Adenoids are normal in children
especially between the ages of 68 years. Most of them are
asymptomatic and can be left alone. Enlarged adenoids can obstruct
the nasopharynx and ET leading to mouth breathing, snoring, apnea
and glue ear. Adenoids should only be removed if they are large
and symptomatic. Nasopharyngeal CA on the other hand, originates
usually from the lateral aspect of nasopharynx (FOR) and rarely seen
in children.
63. a. This is an image of the right middle meatus. It shows an oval-shaped
opening in front of the outline of the uncinate process. There is
no polyp or mucopus noted and the OMC area appears free from
any obstruction.
76
b. An accessory maxillary ostium. This ostium is usually located more
anteriorly and found in front of the uncinate process (it can be
multiple) as compared to the natural ostium, which is sited higher
and posterior to it; thus unusual to be seen directly on viewing
using 0 endoscope. It can be the passage for an antrochoanal
polyp as it exits from the antrum before heading posteriorly toward
the choana. Physiologically, mucociliary function and drainage of
the maxillary sinus is directed toward natural ostium. If an ostia
is noted without the uncinate being removed, then it is almost
always an accessory ostia.
64. a. Image showed absence of entire left inferior turbinate with the
exception of its posterior end. Mucous membrane covering the
inferior concha remains. Opening of the nasolacrimal duct is
clearly seen and appears patent. There is a furrow and buckling
of the septum toward the opposite site likely caused by DNS. The
middle turbinate and nasopharynx can be seen.
b. Subtotal inferior turbinate trimming or turbinectomy. It was
probably done to relieve persistent nasal obstruction caused by
compensatory inferior turbinate hypertrophy secondary to the
DNS.
c. Perioperative bleeding can be significant. This is especially
so when its posterior end is encroached upon (branches from
sphenopalatine vessels). Thus, postoperative nasal packing is
necessary. This procedure should not be performed in cold
countries to avoid crustings and possible development of
secondary atrophic rhinitis. Presently, alternative procedures
are available ranging from superficial cautery, radiofrequency,
powered instruments, or even laser. These treatment options need
to be discussed with the patient and the procedure chosen should
cause minimal morbidity with lasting effects.
65. a. This patient appears to have a totally reduced airway on the left
side as demonstrated by the cold spatula test.
b. Endoscopic nasal examination revealed a mass along the floor of
the left nasal cavity below the middle turbinate.
c. A history of recurrent epistaxis in a boy should alert the clinician to
the diagnosis of angiofibroma and above-mentioned findings would
clinically confirm angiofibroma. Management consists of confirming
the diagnosis with a CT contrast, where a blush is noted due to the
vascularity of the lesion. The tumor extents are noted as it can involve
the pterygopalatine fossa or even extend intracranially. Treatment

Rhinology
is usually by surgical excision either endoscopically or via open
traditional approach after an embolization procedure.
66. a. The images reveal evidence of a post endoscopic dacryocystorhino-
stomy (DCR), which is usually performed for patients with
epiphora. The ophthalmologists usually flush the canaliculi and
if this fails to relieve epiphora, then the option of DCR is offered 77
to the patient. A silicon tune (ODonoghue or Jones) is inserted
into both the upper and lower canaliculi and delivered into the
lacrimal sac and secured intranasally.
b. The tube is kept for an average of 3 months and then removed in
an outpatient setting. In our experience, a success rate of 94% can
be achieved this way.
67. a. The PNS CT scan shows the left sphenoid sinuses filled with soft
tissue density mass or fluid that displaces the anterior sphenoidal
wall. The bony margins are thickened and ossified indicating the
possibility of osteogenesis.
b. The possible diagnosis includes left sphenoidal mucocele, polyps,
sinusitis, fungal sinusitis or sphenoidal tumor.
c. The patient will require an endoscopic examination, especially of
the sphenoid-ethmoid complex. The management would surgically
intervene, where sphenoidotomy will be required to excise and
examine the origin of the mass. Biopsy is mandatory.
68. a. The nasal endoscopic examination reveals a crusted lesion covered
with mucopus in the left nasal cavity. The PNS CT scan reveals
left nasal cavity mass with calcification.
b. The diagnosis is left nasal cavity rhinolith.
c. The etiology is the formation of calculi in the nasal cavity, which
Image-Based Case Studies in ENT and Head & Neck Surgery

usually occurs around the nucleus of a small exogenous foreign


body, blood clot or impissated secretion by slow deposition of
calcium and magnesium salts.
d. The patient should undergo removal of the rhinolith under
sedation. Later, she needs to perform regular nasal douche and
take antibiotics to cover infection.
69. a. The image reveals a childs face with a bluish discoloration noted
at the left alar of the nose.
b. The most likely diagnosis is left alar hemangioma or arteriovenous
malformation.
c. Most of this lesion can be treated conservatively as the lesion will
regress with age. If there is no regression, we can consider medical
treatment like propranolol or give option of surgery to excise the
lesion.
70. a. This photo shows a middle-aged gentlemans face with a mid nasal
mass located at its right side wall encroaching the midline. The
edges appeared elevated with early formation of ulcer evident by
shallow central slope and scab formation.
b. The provisional diagnosis is basal cell carcinoma (BCC) and the
differential diagnoses are squamous cell CA and verrucous CA.
c. This patient needs to undergo a wide local excision of the lesion
with clear margin. Later, a rotation flap can be harvested to cover
the excised skin.
71. a. The X-ray shows a nasal bone fracture.
78 b. The fracture can be fixed with closed reduction either under local
anesthesia infiltration or under GA.
c. Other lesions that should not be missed include septal hematoma,
craniofacial fractures, and intracranial injuries.
72. a. This endoscopic image shows a synechia of right nasal cavity with
an adjoining adhesion between right nasal wall and septum.
b. The diagnosis is post endoscopic sinus surgery synechiae.
c. He can be treated either conservatively or surgically. As he is well
and without nasal symptoms, wait and see policy can be applied
here. However, if the nasal symptom occurs and proved related to
this scar tissue, its excision is then recommended with temporary
insertion of intranasal splint for about a week.
73. a. The left photo shows red and swollen external nose with deviation
of dorsum of the nose and the right nasal endoscopic view shows
a swelling involving the superior part of the vestibule.
b. The possibilities are extrusion of implant in the nose, and cellulitis
of the external nose, as well as foreign body granulomatous
reaction.
c. This patient should be admitted and start parental antibiotics. The
extruded and infected implant should be removed and immediate
surrounding tissue biopsied.
74. a. This lady has bilateral conjunctival chemosis, lids edema and
redness of the eyes.
b. The possible diagnosis is cavernous sinus thrombosis resulting
from direct upward spread of the recurrence tumor and invasions
of the cavernous sinus.
c. The prognosis is poor.
75. a. The nasal endoscopic images show a glistening polypoidal
mass with a smooth surface in right nasal cavity and extending
posteriorly to the nasopharynx. The most likely diagnosis will be
antrochoanal polyp.
b. A PNS CT scan should be ordered to assess the origin and extent
of the polypoidal mass. Later, the mass should be removed and
sent for HPE.
76. This image shows a gentleman with right nasal alar skin lesion (likely
BCC) with planning of glabellar/forehead rotation skin flap. This flap
will be excised and sutured once taken well by the recipient site and
viable.

Rhinology
79
3 Head and Neck

Questions

Q. 77. This patient developed a gradual painless swelling in front of the


left ear. She did not have any other complaints.
a. What are the possible diagnoses?
b. What is the subsequent management?

Q. 78. This patient had superficial parotidectomy performed.


a. Name the structures as numbered.
b. What are the landmarks that can be used to identify facial nerve
intraoperatively?
Head and Neck
Q. 79. This patient complained of recurrent sore throat associated with
fever. What is the diagnosis and subsequent management?

81

Q. 80. This patient presented with high fever for 3 days duration,
associated with severe odynophagia and inability to tolerate
orally. Neck examination revealed multiple tender cervical
lymphadenopathy.
a. Describe the findings on throat examination.
b. What is the diagnosis?
c. What complications can occur?
d. How would this patient be managed?
Image-Based Case Studies in ENT and Head & Neck Surgery

Q. 81. An elderly lady was admitted with odynophagia after a meal.


Examination revealed pooling of saliva in the hypopharynx, upon
which a lateral neck X-ray was ordered.
a. Describe the X-ray findings.
b. What is the subsequent management?

82

Q. 82. This 14-year-old girl complained of slow growing painless swelling


over left anterior triangle of the neck. Computed tomography scan
showed a well-defined hypodense cystic lesion.
List the possible etiologies and its subsequent management.
Q. 83. This 40-year-old lady presented with dysphagia to solid foods
since past 2 years. No loss of appetite or weight loss was noted.

Head and Neck


Investigations showed Hb: 84.7 g/l, MCV: 72 fl, MCH: 21.1 pg,
MCHC: 294 g/l, serum ferritin: 7.2 mg/l, serum iron: 2.35 mmol/l.
a. Describe the barium swallow findings.
b. What is the diagnosis?
c. How would this patient be managed further?

83

Q. 84. What is shown and what are its indications?


Image-Based Case Studies in ENT and Head & Neck Surgery

84

Q. 85. This patient presented in the ENT clinic with complaints of


bilateral parotid swellings and hand problems.
a. What is the diagnosis?
b. Why do these patients present with the ENT clinic?
c. What does the picture of her lips indicate?
A

Head and Neck


85

Q. 86. This child presented with recurrent purulent discharge from the
lower neck near midline. Neck exploration was performed after
the infection settled and specimen as shown was obtained.
a. What is the diagnosis?
b. Where else in head and other areas similar lesion can occur?
Image-Based Case Studies in ENT and Head & Neck Surgery

Q. 87. Left radical neck dissection was performed in a patient with


metastatic cervical lymphadenopathy from carcinoma of larynx.
Redivac drain and collection bottle is shown above.
a. What is the likely diagnosis?
b. How would this patient be managed further?

86

Q. 88. This patient presented with a reduced hearing and mild nasal
speech. Oropharyngeal findings were as shown.
a. What is the diagnosis?
b. What is the likely cause of his reduced hearing in relation to this
finding?
c. How would this patient be managed further?
Q. 89. This is an intraoperative finding during a routine surgery for a

Head and Neck


parotid swelling.
What is the diagnosis and the subsequent management?

87
Image-Based Case Studies in ENT and Head & Neck Surgery

88

B
Q. 90. This patient presented with multiple firm swellings of various
sizes involving the skin of the whole body. He also has a bigger
neck lump on the right side of his neck.
a. What is the diagnosis?
b. What are the other manifestations of the disease in the head
and the neck area?
A

Head and Neck


89
B
Q. 91. The above patient had an operation for a lesion involving the
right side of his tongue. On follow-up, oral cavity and forearm
examinations revealed the above findings.
a. With what condition this patient would have suffered prior to the
operation?
b. What operation was performed and its indication?
c. What other options are available for similar lesion?
Image-Based Case Studies in ENT and Head & Neck Surgery

Q. 92. This picture was taken during an oncologic surgery.


What is being performed and under which circumstance the
procedure is necessary?

90

Q. 93. This is an intraoperative image of a child, who presented earlier


with a painless central neck swelling.
a. What is shown?
b. Give the diagnosis.
c. Briefly describe the operative procedure.
A

Head and Neck


91

B
Q. 94. A 54-year-old female with noninsulin dependent diabetes mellitus
presented with severe odynophagia and fever since 4 days prior
to admission. She denied having difficulty in breathing or stridor.
On examination, she appeared dehydrated and the neck was
swollen. Random blood sugar test showed reading of 18 mmol/l.
Imaging studies were ordered as shown.
a. Describe the findings of both radiological images.
b. What is the likely diagnosis?
c. Outline further management of this patient.
Image-Based Case Studies in ENT and Head & Neck Surgery

92

B
Q. 95. This image was taken during an oncologic surgery for tongue
carcinoma. The submandibular gland was retracted downward
on the left and the opposite view at the completion of surgery.
a. What is the likely surgery performed?
b. Identify the structures as numbered.
Head and Neck
Q. 96. This gentleman presented with recurrent painful tongue lesions
which usually lasted for few days.
a. Describe the lesions.
b. What is the diagnosis?
c. How would you manage this patient optimally?

93

Q. 97. This is the image of the tongue of a female patient who presented
with complaint of blood stained to frank bleeding from the mouth.
a. Describe the lesion shown.
b. What is the most likely diagnosis?
c. Describe the methods for its removal and justify the reasons.
Image-Based Case Studies in ENT and Head & Neck Surgery

94

Q. 98. A 21-year-old lady was involved in a car accident and sustained


multiple injuries involving the face and lower limb. Oral cavity
examination was done and X-ray ordered. Her limb injuries were
managed by the orthopedic team.
a. Describe the findings of both the images.
b. Give the diagnosis.
c. What other injuries should not be missed?
d. Outline further management of this patient giving focus on
the facial injury.
Head and Neck
Q. 99. This is the appearance of the floor of mouth.
a. Describe the findings and give the diagnosis.
b. Should it be treated?

95
Image-Based Case Studies in ENT and Head & Neck Surgery

96

Q. 100. This lady presented with fever and swollen neck of few days
durations. Blood test showed leukocytosis and culture showed
no growth.
a. What is the diagnosis?
b. How is this condition managed?
Head and Neck
Q. 101. These instruments were designed to be used for oro-and hypo-
pharyngeal procedures.
a. Name the instruments.
b. What are its clinical uses?

97

Q. 102. This female patient presented with a painless neck swelling. A


diagnostic surgery under local anesthesia was performed.
a. What type of surgery is being demonstrated?
b. Give its indications.
c. Name an alternative technique and its justification.
Answers
Image-Based Case Studies in ENT and Head & Neck Surgery

77. a. Painless preauricular swelling includes parotid tumors and


metastatic lymph nodes. The majority of these tumors are benign
particularly pleomorphic adenoma and its growth is slow and
gradual; thus no discernable pain will be noted as in this patient.
However, the presence of pain or facial nerve palsy would favor a
malignant tumor, e.g. high-grade mucoepidermoid tumor, adenoid
cystic carcinoma (CA) or otherwise a malignant transformation
from longstanding pleomorphic adenoma (rare).
b. Fine needle aspiration cytology (FNAC) would determine the
nature of swelling, especially in differentiating, whether it is benign
or malignant. Computed tomography (CT) scan will delineate
the extent of involvement by tumor. Superficial parotidectomy
with preservation of facial nerve (with aid of nerve monitor or
stimulator) would be carried out for pleomorphic adenoma. For
malignant tumor, total or extended parotidectomy with or without
postoperative radiotherapy should be performed depending on
its extent. Patient need to be informed of facial nerve palsy as the
nerve may need to be sacrificed if involved by tumor.
78. a. 1. Facial nerve trunk
2. Temporozygomatic branch
3. Cervicofacial branch
4. Buccal branch
98 5. Marginal mandibular branch
6. Cervical branch
7. Posterior belly of digastric
8. Internal jugular vein
9. Sternomastoid muscle (retracted laterally)
10. Deep lobe of parotid.
Arrow points to mastoid tip area.
b. Several landmarks can be used to identify facial nerve; namely
tragal pointer, mastoid process, posterior belly of digastric, and
tympanomastoid suture. The nerve is 1 cm deep and inferior to
tragal pointer (the tragal cartilage end); the nerve passes downward,
forward and laterally, immediately above the upper border of the
posterior belly of digastric, lateral and slightly posterior to the base
of styloid process; the nerve bisects the angle between the two
pieces of bone (the anterior border of the mastoid process and the
vaginal process of tympanic bone) at the tympanomastoid suture.
In difficult cases, the peripheral branch can be traced upto the
main nerve trunk (retrograde approach).
79. There is bilateral tonsillar hypertrophy (Grade II-III) (Grade I no tonsillar
enlargement, Grade IV kissing tonsils). The history points to chronic
tonsillitis in which there is recurrent infection with normal phase
in between. Treatment depends on the frequency of infection and
morbidity experienced by patient. Tonsillectomy is advised when the
attack is too frequent and in those requiring frequent medical leave
from work or school. In addition, this would relief snoring in some
patients, who have obstructive symptom related to its enlargement.
80. a. There is a thick mucopus emanating from the left tonsillar crypts
with an adherent membrane of similar nature coating the right
tonsil and the posterior pharyngeal wall. The tonsillar pillars and
uvula were inflamed and edematous.
b. This finding is typical of acute follicular tonsillitis. However, the
possibility of infectious mononucleosis should be considered based
on the findings of multiple cervical lymphadenopathy.
c. Dehydration with electrolytes imbalance, septicemia, and scarlet
fever. Patient with infectious mononucleosis may develop rashes
upon taking ampicillin.
d. In view of poor oral intake and dehydration, this patient needs
hospital admission. Intravenous fluid and antibiotic are given until

Head and Neck


the patient can tolerate better orally. Full blood count will show
leukocytosis with a left shift in bacterial tonsillitis and lymphocytosis
in viral tonsillitis including infectious mononucleosis.
81. a. X-ray showed a sharp radiopaque shadow in cervical esophagus
opposite C6/C7 cervical vertebrae. There is no air column or
significant prevertebral soft tissue swelling, which will also indicate
the presence of foreign body. This would be most likely due to
foreign body impaction particularly fishbone, which is a common
99
dish served in Southeast Asian countries.
b. The patient needs an esophagoscopy examination under anesthesia
to remove the foreign body. If the patient is unfit to undergo general
anesthesia (GA), removal can also be done under sedation in
endoscopy suite using flexible esophagoscopy. Nasogastric tube
should be inserted if the mucosa lacerated and postoperatively the
patient monitored for signs of complications like fever, tachycardia,
and central chest pain.
82. The swelling appears to be located at the carotid triangle of the
neck and anterior to sternomastoid muscle at its junction of upper
one-third and lower two-third. The cause of swelling here would
include cervical lymph node, non-Hodgkin lymphoma, branchial
cyst, and carotid body tumor. Considering the patients age and
clinical presentation, branchial cyst would be the most likely
diagnosis. It appears as a well-defined cystic lesion anteromedial to
the sternomastoid muscle. It is the most common type of branchial
arch anomaly and usually present in young age patient. Computed
tomography scan revealed a homodense well-defined rounded lesion.
Treatment is by the complete excision of the cyst.
83. a. The barium swallow revealed a persistent anterior narrowing defect
Image-Based Case Studies in ENT and Head & Neck Surgery

opposite the level of C5/C6 vertebrae.


b. This is most likely due to an esophageal web. The clinical feature
of dysphagia and investigation results revealed hypochromic
microcytic anemia, which is typical in this condition.
c. There is a small risk of malignancy associated with the web.
Thus, a thorough esophageal examination should be carried out.
Usually, the esophageal web appears as a crescentic narrowing just
at/below the cricopharyngeal sphincter. Therefore, attention must
be paid at this area and examination should be proceeded slowly.
Biopsy should be taken and followed by dilatation procedures.
Iron deficiency anemia need to be corrected and dietary advice
given. This condition is also known as Paterson-Kelly and Plummer-
Vinson syndrome.
84. These are esophageal bougies used for dilating strictures. Usually,
the smallest size that fits into the narrowing be used first followed
by subsequent sizes, until adequate dilatation achieved. Dilatation
should be stopped if there is bleeding as this would indicate a
significant injury to submucosal layer that would predispose to further
fibrosis and that will worsen the stricture. The bougies may need to
be chilled first to stiffen its tip for the ease of insertion. In addition,
this would also help to reduce postprocedural mucosal hyperemia
and edema. Care is taken not to overdilate in a single session as to
reduce the risk of tear or perforation. Repeated procedure may need
to be done depending on the patients relief of symptom.
100 85. a. These pictures are characteristic of rheumatoid arthritis. They
have joint pain and demonstrate the classical finger deformities
as shown in the picture.
b. Patients usually present to ENT clinic with dryness of mouth and
eyes, and neck swellings are typically grouped under secondary
Sjogrens syndrome.
c. The picture of lip indicates a sublabial biopsy that is usually
performed to confirm the diagnosis of Sjogrens syndrome, where
typical lymphocyte infiltration is seen.
86. a. The provisional diagnosis would be a branchial fistula/sinus. This
characteristically opens at the inferior part of the neck just anterior
to sternocleidomastoid muscle. It is due to an embryological
anomaly due to incomplete fusion of branchial cleft. Patients
usually present with recurrent discharge from the fistula/sinus.
This track usually extends upward between the bifurcations of
the carotid arteries and subsequently ends in either the lower part
of tonsil or pyriform fossa depending on which branchial cleft is
involved. Management is via surgical excision where the entire
tract is removed totally.
b. Similar fistulas can occur according to the branchial cleft/arch
involved. First arch fistulas can open into bony-cartilaginous
junction part of ear canal while the second arch fistula can open
in upper neck and traverses through the parotid and facial nerve.
87. a. The cumulated secretion revealed dual layered color with the lower
one hemoserous and the upper milky white. This is typically seen
in chylous leak in which the thoracic duct has been severed as it
entered the brachiocephalic vein in the lower part of neck on the
left side. As homeostasis secured in the next few days the content
of the bottle would be completely creamy white in color.
b. This complication can be managed conservatively initially by
giving nothing by mouth but putting the patient on intravenous
fluid (Note: fatty food will make it worse). However, this would
be insufficient for the patient postoperatively (catabolic state and
tissue healing phase); thus total parenteral nutrition will be usually
necessary. Immediate exploration is rarely needed, as in majority
of the cases it will be resolved spontaneously, unless it is a high
output leak. Care must be taken during neck dissection on the
left side and the presence of clear fluid near the lower anterior
neck would indicate severed thoracic duct that should be repaired

Head and Neck


immediately. In some cases of persistent leak, radiotherapy can
be started early and this will seal the leak.
88. a. A bifid uvula.
b. The possibility of submucous cleft palate should be considered.
Thus, the palate needs to be palpated carefully to look for
the defect. This condition may predispose to Eustachian tube
dysfunction and glue ear, especially in children. Adenoid removal
is to be avoided as it may worsen the ear problem. In this patient, 101
no submucous cleft was found and the ear showed mild retraction
bilaterally without evidence of effusion.
c. Audiometry and tympanometry need to be done to assess the
hearing loss and tympanic membrane compliance. Valsalva
maneuver is to be advised, and Myringstomy and grommet
insertion can be considered if the hearing loss is significant.
89. There appears to be a swelling attached to three distinct cord-like
branches, most probably facial nerve as this is a parotid surgery. The
swelling is most likely to be a facial nerve schwannoma, which is a
benign tumor of the facial nerve. This can be found in other nerves
in the body and can be malignant in some cases. These patients
usually do not have a facial nerve palsy during early stages and
present with a parotid swelling. Fine needle aspiration cytology is
useful but is not foolproof. It is often only discovered during surgery
and thus it is needed to inform every parotid surgery patient of the
small but real chance of facial nerve schwannoma. The management
consist of excision of the tumor with adequate margin (confirmed
with frozen section) and as far as possible, nerve grafting (sural or
greater auricular according to surgeons preference) in the same
sitting. Recovery is quite good but the patient must be warned that
recovery can take months (6 months and more) and the facial nerve
function will never be back to normal; usually House-Brackmann
Image-Based Case Studies in ENT and Head & Neck Surgery

Grade IIIIV. Recurrence of tumor is possible and thus a negative


frozen section of the margin is important.
90. a. This appears to be neurofibromatosis or von Recklinghausens
disease. This is an autosomal dominant hereditary disorder, in
which patients have multiple neuromas that can present anywhere
in the body. In rare instances, it can be due to spontaneous
mutation. Other features of this condition include caf-au-lait spots
and plexiform fibroma.
b. Clinical presentation depends on the site of the neuroma. In this
patient, the neck swelling is probably due to a neuroma in the
parapharyngeal space. Involvement of cranial nerves or in the spine
can result in fatal outcome. Bilateral vestibular schwannomas are
atypical presentation, where patients present with hearing loss and
imbalance due to brainstem compression. Management consists
of excision of tumors that are symptomatic.
91. a. This patient had a radial forearm free flap (Chinese free flap) that
is usually used to reconstruct defects in head and neck areas in
ENT specialty.
b. Considering the lateral tongue as the area of primary excision, this
would most probably be due to the tongue CA usually squamous
cell. As the area of excision in the tongue appears to be less than
half, then it is very likely to be a partial glossectomy. The degree of
tongue excision depends on the size of the primary lesion with a
102 good tumor-free surgical margin. Thus, it can be a hemiglossectomy
(half of tongue), subtotal or total glossectomy depending on the
size of primary lesion. Neck dissection is also carried out in cases
of clinically or/and radiologically proven neck metastases.
c. Early cases without neck node metastasis can be treated with
radiotherapy.
92. This is a picture that shows the elevation of a deltopectoral flap. It
is a pedicled flap that is based on the upper four perforators of the
internal thoracic artery and used quite frequently in reconstruction of
defects in head and neck oncologic surgery, especially in oral cavity,
cheek and floor of mouth. The main disadvantage is that it entails a
two-stage procedure that requires a staged release of the flap upon
the take up of the recipient site after several weeks.
93. a. This photo shows an intraoperative view of removal of a midline
anterior neck cyst below the hyoid bone.
b. This appears to be a thyroglossal cyst based on the location of the
lesion.
c. This operation is called Sistrunks operation. A transverse midline
skin incision is made over the swelling and subplatysmal flap is
raised. The lesion is mobilized from the surrounding structures
until its neck is identified, which usually runs through the hyoid
bone. The hyoid body is then excised and the tract is followed
usually until it ends at the base of tongue.
94. a. The lateral neck X-ray reveals widening of retropharyngeal space and
the CT scan reveals a hypodense lesion at the retropharyngeal space.
b. This appears to be a retropharyngeal abscess.
c. She needs to be admitted for hydration, to control her blood
glucose with insulin sliding scale, intravenous antibiotics and
incision and drainage of the abscess under GA.
95. a. This appears to be a selective neck dissection most probably a
supraomohyoid neck dissection with wide excision of tongue CA.
b. A: Digastric muscle
B: Mylohyoid muscle
C: Internal jugular vein
S: Sternomastoid muscle
M: Mastoid tip area
SM: Submandibular gland

Head and Neck


Star: Digastric tendon
Black arrow: Hypoglossal nerve
White arrow: Lingual nerve.
96. a. There are multiple ulcers noted at ventral surface of the tongue as
well as the floor of the mouth.
b. The diagnosis is aphthous ulcers of the oral cavity.
c. She needs oral gargle with lignocaine and triamcinolone gel (e.g. 103
oral aid) to reduce the pain and promote the healing process.
97. a. The image shows a bluish-red fungating swelling arising from right
paramedian aspect of the distal anterior two-third of the tongue.
b. The most likely diagnosis is tongue hemangioma.
c. The mass should be removed using coagulation diathermy to
coagulate its base and then removed in the same setting. This
would reduce intraoperative bleeding. Alternatively, bipolar
radiosurgical device can be used such as Surgitron with the
parameter set at cut and coagulation mode.
98. a. The images showed discontinuity of the lower teeth and the
alveolus with minor bruise of inner aspect of the lower lip.
b. The most likely diagnosis is close fracture of the mandible
(parasymphysis).
c. Other injuries should not be missed are temporomandibular joint
injuries, e.g. effusion or dislocation, other craniofacial injuries,
intracranial injuries and neck injury especially involving the
airway.
d. Further management of this patient focusing on the mandibular
injury is open reduction with wire fixation of the fractured segment.
Analgesics and anti-inflammatory medication need to be given
Image-Based Case Studies in ENT and Head & Neck Surgery

adequately.
99. a. The image shows multiple rounded lesions arising from the inner
aspect of the mandible. The gingiva and teeth are normal. The
diagnosis is torus mandibularis which is a slow-growing benign
bony mass.
b. This lesions can be left alone as it is usually asymptomatic and
found by chance.
100. a. The most likely diagnosis is Ludwigs angina.
b. She should be admitted and started on parenteral antibiotics.
Beware of the upper airway obstruction as the submental swelling
will push the tongue backward and obstruct the upper airway.
Intubation or tracheostomy can be performed to relieve the upper
airway obstruction. Incision and drainage can be performed if
abscess is formed. A transverse incision extending from one angle
of mandible to the other is made with vertical opening of the
midline musculature of tongue by using a blunt hemostat.
101. a. These are angled biopsy forceps (the upper two instruments)
with an upturn and sideway biting angles used typically for
hypopharyngeal area. The lower black-colored instrument is
Kaluskar forceps which is widely used for the removal of foreign
body at tongue base area.
b. These instruments are used clinically for removal of foreign body
and taking biopsy.
102. a. The surgery being performed is excisional lymph node biopsy.
104 b. It is indicated to confirm the diagnosis by histopathological
examination usually necessary in suspected lymphoma.
c. The alternative technique is FNAC as this can be performed
as office clinic procedure with 8090% sensitivity (operator
dependent).
4 Laryngology

Questions

B
Q. 103. An immigrant lady presented with the history of hoarseness
preceded by chronic cough. Laryngeal examination was performed
and a chest X-ray ordered. Mantoux test was negative.
a. Describe the laryngeal lesion as shown.
b. Describe the chest X-ray findings.
c. What is the most likely diagnosis?
d. How would this patient be managed further?
Image-Based Case Studies in ENT and Head & Neck Surgery

106

B
Q. 104. A young Malay lady presented with sudden onset of hoarseness
and stridor after ingesting tablet containing evening primrose oil.
Laryngeal endoscopic view and lateral soft tissue X-ray attached.
a. Describe the X-ray findings.
b. Explain the endoscopic findings.
c. What is the likely diagnosis?
d. How would this patient be managed further?
Q. 105. This is an intraoperative view of a patient, who is undergoing a
total thyroidectomy.

Laryngology
a. Describe what is being highlighted.
b. What would happen if this is severed or damaged and how
to avoid this injury?

107

Q. 106. This is the neck image of a patient, who had his larynx removed
for the recurrence carcinoma of larynx.
a. Describe the findings.
b. What is needed to be considered for this patient?
Image-Based Case Studies in ENT and Head & Neck Surgery

108

B
Q. 107. This patient presented with a chronic breathy voice after thyroid
surgery performed many years earlier. Videostroboscopy was
performed and still images taken during phonation are enclosed.
a. Describe the findings and likely diagnosis.
b. How would this patient be managed further?
Q. 108. A 9-year-old school boy presented with history of hoarse voice
for 3 years duration.
Examination under anesthesia was performed and findings were

Laryngology
as shown.
a. Describe the findings.
b. What is the likely diagnosis?
c. What is the subsequent treatment?

109

Q. 109. This patient presented with the hoarseness of voice for few months
duration. He gave the history of chronic smoking earlier, but no
alcohol consumption noted. Direct laryngoscopy was performed
and intraoperative image was as shown.
a. Describe the findings.
b. What is the likely diagnosis?
c. How would this patient be managed further?
Image-Based Case Studies in ENT and Head & Neck Surgery

110

B
Q. 110. A 1-year-old child presented with worsening stridor and difficulty
in breathing for which emergency tracheostomy was performed.
Computed tomography scan was ordered and subsequently the
child scheduled for direct laryngoscopy. He was born prematurely
at 34 weeks and managed in pediatric intensive care unit, after
which he was discharged well.
a. Describe the findings on direct laryngoscopy and the
Computed tomography scan findings.
b. What is the diagnosis?
c. How would this child be managed further?
A

Laryngology
111

B
Q. 111. This young patient presented to outpatient ENT clinic with a
feeling of food sticking in the throat and fluctuating voice change
usually worse in the morning. There were no nasal symptoms
or recent respiratory tract infection. Videostroboscopy was
performed and still images on adduction and abduction were
shown, respectively.
a. Describe the findings.
b. What is the most likely diagnosis?
c. How would this case proceed?
Image-Based Case Studies in ENT and Head & Neck Surgery

Q. 112. This is the direct laryngoscopic view of a patient, who presented


with hoarseness of voice. Describe the findings and its clinical
implications.

112

Q. 113. A 9-year-old prefect schoolboy presented with persistent change


of voice over 6 months duration. On duty, he gave instructions
and sometimes needed to talk louder than usual.
a. Describe the findings.
b. Give the diagnosis.
c. How would this boy be managed optimally?
d. What is the role of surgery in this case?
Q. 114. A schoolteacher presented with hoarse voice for 8 months
duration, when she posed difficulty in teaching. She never had
similar complaints before. On consultation her voice was raspy,
and endoscopic evaluation showed no evidence of vocal fold
palsy. Still image was as shown.

Laryngology
a. Describe the findings.
b. What is the diagnosis?
c. Is there any role of speech therapy in this case?
d. How would this lesion be treated?

113

Q. 115. A chronic smoker presented with severe hoarseness for about


an year duration. On consultation, his voice was grave in
nature and has limited projection. Endoscopic assessment and
microlaryngeal surgery was performed.
a. Describe the intraoperative findings.
b. What is the diagnosis?
c. Describe the definitive procedure that needs to be performed.
d. How would this patient be advised postoperatively with regards
to the prognosis?
Image-Based Case Studies in ENT and Head & Neck Surgery

114

C
Q. 116. These images showed surgery being performed for voice
improvement in adductor vocal fold palsy.
a. What thyroplasty technique is being shown?
b. List the advantage of this surgery as compared to others.
c. What potential complications can occur?
d. Name the injection instrument being used.
Q. 117. A child was intubated due to upper airway obstruction for 10

Laryngology
days duration. Intraoperative laryngeal assessment was performed
prior to definitive surgical intervention.
a. Describe the findings.
b. What is the diagnosis?
c. List the sequelae if this lesion is not being managed optimally.

115
Image-Based Case Studies in ENT and Head & Neck Surgery

116

B
Q. 118. This is an intraoperative image of a 3-week-old infant, who
presented with stridor. Figures shown were the pre- and
postoperative views, respectively.
a. Describe the findings.
b. Give the diagnosis.
c. What surgery likely was performed?
A

Laryngology
117

B
Q. 119. a. Describe the findings of these images.
b. Give the diagnosis.
c. How would this patient be managed further?
Image-Based Case Studies in ENT and Head & Neck Surgery

118

Q. 120. An elderly patient presented with progressive dysphagia to both


solid and liquid. A radiological investigation was performed but
he developed procedural complication.
a. Describe the findings of the radiological images.
b. What is the diagnosis?
c. Outline the management.
A

Laryngology
119

B
Q. 121. A 30-year-singer suddenly noted sudden change of voice and
laryngeal irritation while singing. She stopped immediately and
gave rest to her voice. However, her voice did not improve; she
decided to get medical advice a day later.
a. Describe the laryngeal findings and give the diagnosis.
b. How is this condition treated?
Image-Based Case Studies in ENT and Head & Neck Surgery

Q. 122. Compare these two endotracheal tubes and list its differences.
What are the indications for its usage?

120
A

Laryngology
121

Q. 123. a. Describe the findings of these images and give possible


diagnoses.
b. What further investigations should be considered?
c. How would this patient be treated?
Image-Based Case Studies in ENT and Head & Neck Surgery

122

Q. 124. a. Describe the findings of these images in a patient who had


total laryngectomy performed.
b. What are the advantages and disadvantages of this procedure?
Answers
103. a. Laryngeal image showed scattered grayish-white exudates,
ulcerations of the vocal cords near the anterior commissure, and
thickened polypoidal interarytenoid mucosa.
b. The X-ray revealed a typical wedge-shaped opacity affecting the
right upper lobe and apical region, with the trachea remaining the
central. This is typical in bronchopneumonia.
c. Tuberculous (TB) laryngitis is secondary to TB bronchopneumonia.
History of chronic cough, which complicates into hoarseness should
raise a high index of suspicion of TB. Further detailed history like fever,
night sweats, loss of weight, and loss of appetite, as well as possibility
of contact with patient suffering from TB must be obtained. Otherwise,
immunosuppressed states need to be ruled out.
d. Full blood count (FBC), erythrocyte sedimentation rate (ESR),
morning sputum for swab and cultures (x3) and Mantoux test.
In this patient the FBC revealed a highly elevated ESR, positive
sputum for acid-fast bacilli, but negative Mantoux test. Thus, this

Laryngology
patient is highly infectious in an apparently non-reacting Mantoux
called anergy state. Cough etiquette and wearing of suitable face
mask must be followed until sputum culture becomes negative on
treatment. Treatment entails combination of TB drug therapy for a
period of at least 6 months. Contact tracings need to be done by
local health authority and treated accordingly.
104. a. There is a thumb-shaped soft tissue swelling in the area of 123
supraglottic larynx originating from the arytenoid area. The
epiglottis is also swollen as suggested by the thickened opaque
shadow just behind the base of tongue. The air bronchogram does
not show any evidence of subglottic or tracheal narrowing.
b. The endoscopy view showed a markedly swollen epiglottis
and edematous arytenoid mucosa with no obvious erythema or
exudates seen. Pooling of saliva is not seen here.
c. Stridor in this patient occurred at supraglottic level. There
is no fever, significant pain, or purulent exudates to suggest
infection. In view of unfamiliar food eaten before and subsequent
development of symptoms, this would be very likely due to
localized hypersensitivity reaction, e.g. angioneurotic edema.
Supraglottic larynx has a rich lymphatic supply and vulnerable
to angioneurotic edema with fast onset of airway obstruction. If
the lesion occurs repeatedly, possibility of C1 esterase deficiency
needs to be considered.
d. Intravenous steroid, e.g. dexamethasone or methylprednisolone,
and antihistamine would be the first-line of treatment. Endotracheal
intubation is justified, in view of worsening airway caliber, while
waiting for the reactions to subside. Antibiotic is not necessary.
105. a. The thyroid lobe is being retracted to the opposite side. The forceps
point to the left recurrent laryngeal nerve as it ascends through
Image-Based Case Studies in ENT and Head & Neck Surgery

the tracheoesophageal groove heading to the larynx by running


behind the cricothyroid joint.
b. Ipsilateral vocal cord paresis or palsy can occur if this nerve
is inadvertently severed or cut. Even excessive traction or
manipulation may result in temporary loss of function called
neuropraxia. The nerve need to be identified early and preserved
in benign thyroid surgery. A bilateral recurrent nerve injury can
occur in total thyroidectomy. Patient may present with a stridor and
a normal voice. Use of nerve monitor may need to be considered in
some instances, e.g. massive goiter, or in revision thyroidectomy.
106. a. Image shows surgical scars running bilaterally downward across
the sternomastoid and meets at the suprasternal notch area (Gluck-
Sorenson). There is an end tracheostomy in which the upper end of
trachea (after the larynx resected and removed) was anastomosed
directly to the skin for the purpose of breathing.
b. Firstly, the stoma need to be covered as contaminants and
foreign body may enter the larynx causing bronchial irritations
or pulmonary complications. Simple handkerchief or scarf can
be used; otherwise commercialized bip or even stoma protector
with filter can be worn. Care is taken while taking shower and of
course patient can get drowned by swimming. Secondly, since the
larynx was removed some form of speech rehabilitation need to be
considered to enable communication. This has to be informed early
124 to the patient and appropriate counseling and available options
informed by speech therapist. Voice rehabilitation techniques
include esophageal speech, voice prosthesis insertion (e.g. Provox
or Blom-Singer prosthesis, which is termed neoglottis procedure),
or by using a battery-operated artificial larynx, e.g. servox.
107. a. There appears to be right vocal cord immobility most likely
resulted from longstanding vocal cord palsy complicating her
previous thyroid surgery. Bowing of the affected vocal cord and
foreshortening of ipsilateral arytenoid can be observed. There is
a significant phonatory gap noted between the vocal cords upon
adduction giving clue to a more severe degree of dysphonia (very
breathy voice quality) as experienced by this patient. No other
significant pathology noted. Thus, the diagnosis is right vocal cord
palsy in paramedian position.
b. Certain operative techniques can be performed to bring the
paralyzed right vocal cord closer to the functioning contralateral
side to improve voice and prevent aspiration (if any).
The ideal final aim is to have an even adduction of the entire
membranous vocal cord with restoration of normal mucosal
waves upon phonation. These procedure ranges from endoscopic
injection [e.g. using fat or calcium hydroxylapatite (CaHA)] to
increase the bulk of the paralyzed vocal cord, to open thyroplasty
technique with appropriate material inserted (e.g. carved silicone
block, Gore-Tex, etc.) to push the paralyzed vocal cord medially.
In this patient, thyroplasty technique with or without arytenoid
adduction seems appropriate since the gap is moderately large.
1
08. a. There is a smooth cystic swelling (with straw-colored content)
arising from the left membranous vocal cord. The contralateral
vocal cord appears thickened like a sessile polyp.
b. Left vocal cord cyst with reactive right vocal cord lesions (likely
thickening and polypoid reaction) resulting from chronic collisions
caused by the cyst during phonation.
c. The cyst need to be excised/enucleated by using microsurgical
technique. The reactive lesion can be left alone initially and
postoperatively monitored as it may regress spontaneously; failing
which, it can also be excised with prior palpation intraoperatively using
special probe for confirmation of fibrosis. However, single session
microsurgical intervention for both lesions is being advocated by some
with excellent results. Postoperative speech therapy is essential. Small
risk of cyst recurrence exists if its wall is incompletely removed.

Laryngology
1 09. a. Image showed an irregular white lesion along the free edge of
left vocal cord extending from the vocal process of arytenoid
posteriorly and encroaching toward the anterior commissure.
b. This is very likely to be a leukoplakia. Social history of chronic
smoking would raise high suspicion of dysplasia or even malignancy.
c. The examination should proceed further to telescopic assessment 125
of the anterior commissure and subglottic area usually using a
30 or 70 endoscope. Adequate tissue biopsy needs to be taken
for histopathological examination. If the report comes back as
dysplasia, repeated biopsy needs to be done at another setting
determined at follow-up sessions. Subepithelial cordectomy can be
performed if there is no evidence of malignancy found. However, if
the basement membrane invasion is proved, then the patient needs
to be treated as having an invasive carcinoma. In case of clear-
cut malignancy, a definitive treatment should be started (usually
radiotherapy, this is a T1 lesion with a very favorable prognosis).
Alternatively, laser surgery can be performed with equally good
result by a formally trained and experienced surgeon.
1 10. a. Direct laryngoscopy reveals normal vocal cords with narrowing
of the subglottic region. The patency of the airway is markedly
reduced, which explains the presentation of stridor. The sagittal
computed tomography (CT) scan slice shows a tracheostomy in
situ with a shelf-like stenosis below the vocal cords and above the
tracheal stoma.
b. This is a typical finding in subglottic stenosis, which can be
acquired (complication of prolonged intubation).
c. Management consists of securing the compromised airway as the
Image-Based Case Studies in ENT and Head & Neck Surgery

very first step. Thereafter, imaging studies are performed (CT scan or
magnetic resonance imaging) to delineate the location and thickness
of the stenotic component. A thin fibrous stenosis can be excised
with laser, while a much thicker one may require a more aggressive
surgical treatment ranging from excision to cricoid expansion with
costal cartilage or even cricotracheal resection with reanastomosis.
In this child, the recommended initial treatment is likely Carbon
dioxide (CO2) laser excision followed by dilatation procedures.
Topical mitomycin C application can be considered. Open surgery
is advised, when this method fails.
1
11. a. The vocal cord appears to be of normal appearance and mobility;
however, the arytenoids appear to be swollen and edematous. This
typically presents as a sensation of foreign body upon swallowing
or throat discomfort.
b. This is most likely to be due to laryngopharyngeal reflux (LPR) disease.
c. After a thorough examination of the nose and nasopharynx to rule
out chronic rhinosinusitis and postnasal drip, a course of proton
pump inhibitors can be prescribed. As inflammation caused by acid
reflux heals slowly, a longer course is usually given ranging from
3 weeks to 3 months. Ideally, esophageal pH monitoring should
be performed in all cases. However, many laryngologists decided
to treat LPR based on symptoms (reflux symptom index) and the
typical endoscopic findings. Helicobaiter pylori and malignancy
should not be missed.
126 1 12. This image shows a normal left vocal cord, while the right cord
shows a swelling in its mid-membranous portion with an abnormal
vessel traversing it and areas of punctate hemorrhage. The abnormal
vessel is believed to be involved in the pathogenesis of benign vocal
fold lesions including vocal fold cyst and polyp. The normal vocal
fold vessels are much finer and run parallel to the long axis of the
vocal fold instead of transversely and tortuously, as seen in this
image. Bleeding may occur spontaneously or upon voice overuse
and it appears as punctate or subepithelial hemorrhage. Repeated
events predispose to prolonged reparative changes within the lamina
propria layer of the affected vocal cord. Initially, videostroboscopy
shows disturbance of mucosal waves, whilst longstanding untreated
lesions may manifest itself as polyp, cyst or even Reinkes edema.
In addition to the removal of the cyst, this abnormal vessel needs to
be ablated usually using a bipolar diathermy at low power setting
or alternatively by using laser [usually potassium-titanyl-phosphate
laser, or CO2 laser at defocus mode]. Pulsed dye laser is an excellent
alternative for this purpose if available.
113. a. Figure shows symmetric and bilateral nodular whitish swelling
affecting the free edge of the true vocal folds at its mid-membranous
segment.
b. This appearance is typical of mature vocal fold nodules.
c. Management has to take into account the childs behavior, parents
expectation, peer influence, and the rate of voice recovery to
normal. Speech therapy should be the primary treatment in
managing pediatric vocal fold nodules and other benign vocal fold
lesions. Adequate time and encouragement should be given with
the frequency and intensity of voice rehabilitation determined by
the speech therapist as the treatment progresses.
d. Surgical excision is needed as the callus appeared well-formed and
thickened. Intraoperative palpation gives further information on
its consistency. Microsurgical technique is preferred to minimize
disturbance of uninvolved areas of the vocal folds.
114. a. There is a yellowish rounded swelling involving the membranous
left vocal fold with the fold edge appearing evenly bulging
medially. The right vocal fold appeared edematous and its surface
whitish in color. The capillary appeared dilated and running
parallel along the right vocal fold.
b. Left vocal fold cyst with contralateral reactive vocal fold lesion
(likely to be early Reinkes edema with superficial keratoses).

Laryngology
c. Speech therapy is unlikely to resolve the problem, but it should be
targeted at postoperative period for optimal healing environment
while preventing unsupervised vocal use. She should not be
teaching too soon also. Generally, the prognosis of superficial
cyst is favorable and unlikely to recur if completely removed
(enucleated).
d. Microsurgical excision need to be performed followed by 127
postoperative voice therapy.
115. a. Figure shows the laryngeal inlet view of larynx with visible
endotracheal tube. The false vocal folds appeared normal.
However, the true vocal folds were obviously swollen and
glistening with yellowish tinge seen peripherally.
b. This is typical of Reinkes edema or polypoid corditis. In this
condition, there is an accumulation of gelatinous substance like
a gel in the subepithelial layer or Reinkes space. It commonly
occurs in chronic heavy smokers and worsens the voice quality
tremendously.
c. Endolaryngeal microsurgery by using cold instruments is generally
advocated. A more laterally placed linear incision (cordotomy) can
be done by using sickle knife or CO2 laser. Gelatinous material
is then sucked adequately and the epithelium redraped with the
excess trimmed. Tissue glue can be applied at the end, but usually
it is not necessary.
d. Adequate speech voice rest and speech therapy sessions need to
be followed. The patient must quit smoking indefinitely.
1
16. a. Figure shows injection thyroplasty technique by using autologous
fat obtained from the abdominal wall.
b. i. No body rejection
ii. Abundant material and technically not difficult to harvest
Image-Based Case Studies in ENT and Head & Neck Surgery

iii. Direct injection visible under endoscopic view.


c. i. Over injection may cause glottic obstruction and stridor.
ii. Fat resorption overtime and less than optimal vocal fold
approximation.
d. Handheld liposucker for fat aspiration and Bruening syringe for
delivery of fat into vocal fold tissue. Presently, several types of
high pressure injectors are available to be used for fat injection.
1
17. a. Figure shows symmetrical reddish swelling located posteriorly
beside the endotracheal tube corresponding to the cartilaginous
portion of the vocal folds. The false vocal fold and the rest of the
true vocal folds appeared normal.
b. This is most likely due to vocal process granuloma or intubation
granuloma, a misnomer, if related to previous endotracheal
intubation. Too large an endotracheal tube, inadequate sedation,
frequent frictions from insecure tube anchoring, prolonged
intubation, infections, and LPR disease are amongst the common
risk factors implicated for its formation.
c. i. Glottic obstruction due to enlarging granuloma. Stridor can
occur upon extubation with variable dysphonia.
ii. Recurrence after excision. Prophylactic proton pump inhibitor
is usually indicated by most laryngologist.
118. a. Figure shows a significant swelling at the tongue base-epiglottis
junction or so-called vallecula area. It appears cystic and tense
128
with some linear capillaries seen traversing its surface.
b. Vallecula cyst, which is congenital in this case.
c. Uncapping of the cyst is the most likely procedure, which must
be done because much cyst wall together with its covering
epithelium, along the perimeter of the cyst needs to be removed.
Re-epithelialization from the surrounding normal mucosa will
occur upon healing. Initial aspiration allows fluid to be sent for
culture and sensitivity as well as making tissue handling during
surgery easier.
119. a. These figures show a small upper midline swelling in a child which
moves upward upon tongue protrusion.
b. Thyroglossal cyst.
c. Surgical excision is advised as there is a potential risk of recurrence
and infection if treated conservatively. The middle part of the hyoid
bone is removed together with the cyst, also known as Sistrunks
operation.
120. a. These are the images of Barium Swallow study which shows
contrast material along the trachea and lower bronchopulmonary
segments bilaterally.
b. This is diagnostic of barium aspiration.
c. Barium causes inflammatory response along the tracheobronchial
mucosa and pneumonitis. It takes sometimes for the contrast to
disappear naturally but the lung functions need to be monitored
closely if significant aspiration like this one happened. Ward
admission, parenteral corticosteroid and antibiotic cover are
administered. Oxygen saturation (SaO2) monitoring and blood
gases are taken, with oxygen delivered whenever indicated.
121. a. There is bright red appearance affecting the dorsal aspect of the
right true vocal fold beneath its epithelium. This finding is typical of
acute subepithelial hemorrhage which occurs upon voice overuse.
There are no other benign lesions or LPR changes seen.
b. She needs to rest the voice completely over the next 35 days
and thereafter may resume phonation gradually as advised by
the speech language pathologist. She should refrain herself from
singing back too early to allow for normal healing to take place
naturally. Repeated hemorrhage may cause scar changes and
permanent dysphonia if not addressed correctly. The laryngologist
will reassess the vocal fold and look out possible treatable cause
such as vascular ectasia and varix.

Laryngology
122. These are two types of endotracheal tubes used for the purpose of
microlaryngeal surgery. The metallic tube is malleable (flexometallic)
and has two cuffs at its distal end which are filled with colored saline.
The other is normal endotracheal tube with single distal cuff and was
wrapped proximally with shiny reflective tape to conceal the actual
tube.
These are the suitable tubes and their modifications intended to be 129
used for laser microlaryngeal surgery. They prevent the laser beam
striking and puncturing into the endotracheal lumen which can cause
explosion and airway fire. The double cuff offer additional protection
as the more distal cuff can still seal the endotracheal wall if the more
proximal cuff burst.
123. a. These are the endoscopic laryngeal images showing a bluish
swelling with nodular surface alike to mulberry fruit involving
the right supraglottic larynx and encroaching the medial aspect
of ipsilateral pyriform fossa. The appearance suggests a vascular
tumor most likely hemangioma. Depending on patients symptoms
and rate of growth, the possibility of malignant transformations
need to be considered. General examination should rule out similar
lesions elsewhere.
b. Magnetic resonance imaging with contrast would delineate the
extent and vascularity of the tumor especially in an attempt to
identify the blood vessels feeders amenable for embolization if
this is decided to be done.
c. In experienced hand, this can be excised surgically via transoral
approach. Good homeostatic device must be available, e.g. suction
diathermy or insulated bipolar diathermy and the used of suitable
laser, e.g. Nd:Yag or pulsed dye laser. An alternative is via lateral
pharyngotomy approach.
Image-Based Case Studies in ENT and Head & Neck Surgery

124. a. The first figure reveals an end-tracheostomy with a voice prosthesis


in situ (Blom-Singer) while the other shows an endoscopic view
of the pharyngoesophageal segment upon phonation by using the
prosthesis.
b. This is a form of voice rehabilitation to enable phonation after
total laryngectomy. The voice prosthesis gives a better and clearer
voice as compared to oesophageal voice and can be learned
faster. Occlusion of the end-tracheostomy is necessary to shunt
the exhaled air into the pharyngoesophageal segment. However,
innovation and current technology has made hands-free technique
possible, which gives great freedom and boosts the patients morale
tremendously.

130

You might also like